Pathologic Obstetrics Board Review Red

December 6, 2017 | Author: Nicole Santos | Category: Miscarriage, Human Pregnancy, Mammalian Pregnancy, Maternal Health, Diseases And Disorders
Share Embed Donate


Short Description

From FEU 2014 for the PLE. Sorta helpful for rapid review....

Description

ROBERT COLLIER:

HEMORRHAGE IN EARLY PREGNANCY

¢  SUCCESS

¢  CASE

IS THE SUM OF

— 

SMALL EFFORTS,

¢ 

REPEATED DAY IN AND DAY OUT

¢ 

— 

PATHOLOGIC OBSTETRICS BOARD REVIEW

— 

:

35 y/o G3P0 Pregnancy Uterine 12 weeks complains of vaginal bleeding and hypogastric pain. Internal examination revealed closed cervix ,uterus enlarged to 3 months size, adnexae negative for mass or tenderness. WHAT IS THE DIAGNOSIS? WHAT ARE THE DIFFERENTIAL DIAGNOSIS?

Maria Carmen Hernandez-Quevedo, MD Associate Professor B Elizabeth Ahyong –Reyes, M.D. Associate Professor C

ABORTION

INCIDENCE

¢  Pregnancy

¢  80

¢  Fetus

termination before the 20th week born weighs less than 500 grams

¢  TYPES: ¢  1.SPONTANEOUS-

threatened,inevitable,incomplete,complet,missed. Any of these with signs of infection-septic ¢  2.RECURRENT-repetitive spontaneous abortion ¢  3.INDUCED-surgical and medical termination of live fetus

MATERNAL FACTORS

% of spontaneous abortion occur within the first 12 weeks

¢  50%

anembryonic and 50% embryonic

— 

Trisomy(most common) Monosomy X(second most common) Triploidy(molar) ¢  Tetraploidy ¢  Structural anomaly ¢ 

¢ 

¢  5.SURGICAL

¢  9.IMMUNOLOGICAL-antiphospholipid

PROCEDURES-bariatric surgery progesterone if done less than 10 weeks

syndrome

CLASSIFICATION OF ABORTION antibody

VAGINAL BLEEDING,HYPOGASTRIC PAIN 1.THREATENED- cervix closed,uterus compatible with age of gestation ¢  2.IMMINENT-cervix open intact bag of waters.uterus compatible with age of gestation ¢  3.INEVITABLE-cervix open,ruptured BOW,uterus smaller than age of gestation ¢  4.INCOMPLETE-passage of meaty tissues,cervix open,uterus smaller than age of gestation ¢  5.COMPLETE-passage of meaty tissues,cervix closed,uterus small ¢  6.MISSED-cervix closed,uterus not compatible with age of gestation ¢  ¢ 

AOG ¢  10.INHERITED

AND BEHAVIORAL FACTORS ¢  Alcohol,smoking ,caffeine(>5 cups/day)

¢  11.UTERINE

¢  7.OCCUPATIONAL/ENVIRONMENTAL

¢  12.PATERNAL

THROMBOPHILIAS

¢  6.SOCIAL

FACTORS ¢  Arsenic,lead,formaldehyde,benzene,ethylene oxide,DDT(dichlorodiphenylchloroethane)

3.MEDICATIONS- IUD 4.CANCER-radiation(>5 rads) and methotrexate

¢ 

MATERNAL FACTORS

DEFECTS FACTORS

infection

DISORDERS-diabetes,thyroid disorders,celiac disease,anorexia and bulimia, cyanotic heart ,hypertension,inflammatory bowel disease,SLE

Euploid-(50%)normal chromosomes Aneuploid-(50%)abnormal chromosomes

MATERNAL FACTORS ¢  Give

from periodontal disease

¢  2.MEDICAL

¢  EMBRYONIC — 

¢  1.INFECTIONS-Chlamydia,polymicrobial

SEPTIC ABORTION

RECURRENT MISCARRIAGE

ETIOLOGY OF RECURRENT PREGNANCY LOSS

¢  Grp.A

¢  Recurrent

¢  Widely

strep/ Clostridium perfringens or sordelli

pregnancy loss,recurrent spontaneous abortion,habitual abortion

¢  Fever,leukocytosis,hemoconcentration, ¢  hypotension,ARDS,kidney ¢  Broad

failure,DIC

spectrum antibiotics

¢  Completion

curettage

Accepted Causes chromosomal abnormalities

¢  1.Parental ¢ 

¢  Three

or more consecutive spontaneous abortion

¢  Two

or more failed clinical pregnancy confirmed by sonographic and histologic examination(ASRM 2008)

¢  2.Antiphospholipid

antibody syndromeautoimmune disorder(immunity against self)

¢  3.Subset

of uterine anomalies-ashermans syndrome,uterine myoma,mullerian duct anomalies

¢  4.Endocrine

and PCOS

abnormalities-luteal phase defect

CERVICAL INSUFFICIENCY

MANAGEMENT OF CERVICAL INCOMPETENCE

CERCLAGE PROCEDURES

¢  Cervical

¢  Sonography

¢  Mc

incompetence

¢  Painless

cervical dilatation in the second trimester with prolapse of the BOW

¢  Test

for viability and exclude anomalies

for Chlamydia and gonorrhea ¢  Shirodkar-transverse

¢  Cerclage-done ¢  Diagnosed

internal os

incision done at the mucosa

at 12-14 weeks age of gestation

by ultrasound-funneling at the

¢  Transabdominal ¢  Contraindications

to cerclage-bleeding,uterine contractions,ruptured membranes

factors:Cervical trauma due to D and C, conization,cauterization,amputation,DES exposure

Donalds-simpler,pursestring suturing of the cervix

uterine isthmus

cerclage-suture is placed at the

¢  Risk

¢  Types

of cerclage:prophylactic(cervix closed) and rescue cerclage(cervix open)

COMPLICATIONS OF CERCLAGE

ECTOPIC PREGNANCY

OUTCOME OF ECTOPIC PREGNANCIES

¢  Membrane

¢  Ampulla

¢  1.TUBAL

¢  Heterotopic

¢  2.TUBAL

¢  Preterm

Rupture

RUPTURE- earliest (isthmus) later (interstitial)

labor

¢  Hemorrhage ¢  Infection

is the most frequent site followed by the isthmus. Rarest site-previous CS scar pregnancy-intrauterine and ectopic pregnancies coexisting

¢  Highest

risk for tubal implantation-previous tubal surgery

¢  Followed

by sexually transmitted disease- most common chlamydia

ABORTION-fimbrial and ampullary

¢  3.PREGNANCY

FAILURE WITH RESOLUTION

CLINICAL MANIFESTATION

DIFFERENTIAL DIAGNOSIS

DIAGNOSIS

¢  CLASSIC

¢  Abortion

¢  SERUM

TRIAD- amenorrhea,vaginal bleeding,abdominal pain

¢  Infection ¢  Myoma

¢  PELVIC

EXAMINATION-cervical motion tenderness,fullness of the culde sac,tender boggy mass in the adnexa,uterus slightly enlarged

¢  Signs

of diaphragmatic irritation,hypotension,tachycardia,vertigo,syncop e ¢  Passage of decidual cast-endometrium of pregnancy(Arias stella reaction)

with degeneration pregnancy ¢  Ovarian tumor in complication ¢  Salpingitis ¢  Appendicitis ¢  Cystitis,renal stone,gatroenteritis ¢  Molar

DIAGNOSIS

TRANSVAGINAL SONOGRAPHY OF ECTOPIC PREGNANCY

¢  SERUM

¢  Trilaminar

¢  >25

PROGESTERONE

ng/ml-excludes ectopic pregnancy

¢  Complex

endometrium

BETA HCG- discriminatory index more than or equal to 1500mIU/ml expect to see a live intrauterine pregnancy.If no intrauterine pregnancy is seen :failed uterine pregnancy,complete abortion,ectopic pregnancy

¢  DOUBLING

TIME IN NORMAL PREGNANCY-48 hours

LAPAROSCOPY Diagnostic-direct visualization of the Fallopian tube

adnexal masses Therapeutic-surgical management

¢  6cm

¢ 

Slow decline of β HCG

¢ 

Myometrial nodules of hypervascularity by postevacuation ultrasound

¢  Abortion/Tubal

Pregnancies – 25% ¢  Preterm or Term Pregnancies – 25%

—  —  — 

— 

Findings:

Aggressive myometrial invasion Strong tendency to metastasize Irregular bleeding with uterine subinvolution Diagnosed only by persistent elevation of serum hCG in most cases Myometrial Perforation – intraperitoneal hemorrhage Some with lower genital tract metastasis or distant metastasis

HISTOLOGIC CLASSIFICATION OF GTN

INVASIVE MOLE

INVASIVE MOLE

¢  INVASIVE

¢  Formerly

¢  Deep

MOLE

¢  CHORIOCARCINOMA ¢  PLACENTAL

SITE TROPHOBLASTIC TUMOR TROPHOBLASTIC TUMOR

¢  EPITHELIOID

* The diagnosis of GTN is usually based on persistent elevation of serum hCG level without confirmation by pathological study. Management is NOT directed by histologic findings.

called Chorioadenoma destruens ¢  Excessive trophoblastic overgrowth with extensive tissue invasion by trophoblastic cells and whole villi

penetration into the myometrium ¢  Almost always arise from a partial or complete mole ¢  Locally invasive but lacks the tendency for widespread metastasis

GESTATIONAL CHORIOCARCINOMA

GESTATIONAL CHORIOCARCINOMA

¢  Extremely

¢ 

malignant carcinoma of chorionic epithelium ¢  2/3 follow a normal delivery, 1/3 follow molar gestation ¢  Considered anytime there is persistent bleeding after any pregnancy event

Rapidly growing Invades myometrium and blood vesselsà hemorrhage and necrosis ¢  Dark red or purplish, ragged and friable ¢  Early bleeding, sloughing and infection with involvement of endometrium ¢  Dark irregular nodules with involvement of peritoneum ¢ 

GESTATIONAL CHORIOCARCINOMA Involvement of cytotrophoblasts and syncitial elements (one or the other may predominate) ¢  Columns or sheets of trophoblasts penetrating myometrium and blood vessels ¢  Plexiform or disorderly arrangement ¢  No villous patterns ¢ 

GESTATIONAL CHORIOCARCINOMA: METASTASIS ¢  Early

and Hematogenous (affinity of trophoblasts to blood vessels) ¢  Lungs (75%), Vagina (50%) ¢  Vulva, kidneys, brain, bowel ¢  Ovarian theca lutein- cysts seen in over 1/3 of metastatic cases

GTN: CLINICAL COURSE

FIGO 2002 STAGING

¢  Irregular

¢  Stage

bleeding with uterine subinvolution

¢  Bleeding:

continuous or intermittent

¢  Intraperitoneal

hemorrhage: myometrial perforation by invading trophoblasts

¢  May

initially present as vulvar or vaginal metastasis or other distant metastasis

¢  Choriocarcinoma:

fatal without treatment

l- confined to uterus ll-extends outside uterus but limited to genital structures (adnexae, vagina, broad ligament) ¢  Stage lll-extends to lungs with or without known genital involvement ¢  Stage lV-other metastatic sites ¢  Stage

WHO PROGNOSTIC SCORING, 2002

WHO PROGNOSTIC SCORING, 2002

GTN: TREATMENT

GTN: TREATMENT

¢  Best

¢  Subsequent

manged by Oncologist agent chemotherapy: nonmetastatic or low risk disease

¢  Single ¢  ≤

6: low risk disease treatable by single chemotherapeutic agent ¢  ≥7: high risk disease requiring combination chemotherapy

Methotrexate (less toxic) Actinomycin D ¢  Risk

factors for additional chemotherapy beyond first course:

chemotherapy is based on serial hCG levels ¢  Combination chemotherapy for high risk cases: EMA-CO: Etoposide, Methotrexate, Actinomycin D, Cyclophosphamide, Oncovin(Vincristine) Surgery, Radiotherapy

Metastatic disease Single-day methotrexate infusion Complete mole histology

SUBSEQUENT PREGNANCY

SAMPLE QUESTIONS:

¢  Surveillance

¢  1.A

21 y/o G1P0 PU 14 weeks consulted because of minimal vaginal spotting for 2 weeks.No FHT heard on doppler.Cervix closed ,uterus 2 months size,no adnexal mass nor tenderness.What is the type of abortion? ¢  a.Threatened ¢  b.Complete ¢  c.missed ¢  d.incomplete

¢  2.39y/o

¢  4.30

¢  5.A

duration: (minimum)

6 months-molar pregnancy 1 year GTN 2 years if GTN is with metastasis ¢ 

Pregnancy can be achieved after successful treatment

¢  Sonographic

evaluation on early pregnancy and subsequently ¢  Histopathology of placenta or products of conception ¢  Serum β HCG determination 6 week postpartum

¢  3.26y/o

G3P0 PU 14 weeks had an ultrasound finding of live intrauterine pregnancy with funnelling of the internal os.Previous pregnancies were terminated at 4months and 5 months respectively.The BEST management for the patient is ¢  a.bed rest and tocolytic ¢  b.tocolytic and progesterone ¢  c.tocolytic and cerclage ¢  d.cerclage and progestin

y/o G1P0 12 weeks AOG consulted because of vaginal bleeding and foul smelling discharge after an induced abortion.Temp-38 C,cervix open and tender on wriggling,uterus slightly enlarged andtender,adnexa bilaterally tender.What is the BEST sequence of management for this patient? ¢  a.paracetamol , completion curettage,antibiotics ¢  b.broad spectrum antibiotics ,paracetamol, completion curettage ¢  c.broad spectrum antibiotics ,paracetamol,explore lap ¢  d.completion currettage,antibiotics,paracetamol

G4P0 consulted because of recurrent pregnancy losses at 8-10 weeks AOG.She mentioned that her previous pregnancies were all anembryonic.Which of the following is the most possible cause of her miscarriages? ¢  a.chromosomal anomalies ¢  b.endocrine ¢  c.incompetent os ¢  d.infections

35 y/o G4P3 PU 22 weeks has an ultrasound result of IUFD 14 weeks.Cervix is closed and long.Uterus is enlarged to 3-4 months size.Which of the following is NOT part of the work ups of this patient? ¢  a.fasting blood sugar ¢  b.prothrombin time ¢  c.VDRL ¢  d.chest x ray

¢  6.21y/o

G2P0 8 weeks AOG ,Preg test positive,consulted because of vaginal spotting and abdominal pain.cervix closed tender on wriggling,uterus slightly enlarged,with a palpable vauge mass on the right tender on palpation,left adnexa negative for mass and tenderness.Which of the following is the best to arrive at a diagnosis? ¢  a.beta HCG ¢  b.transvaginal ultrasound ¢  c.progesterone assay ¢  d.culdocentesis

¢  7.21y/o

G1P0 5 weeks amenorrhea has a positive pregnancy test and an ultrasound result of thickened endometrium to consider early pregnancy versus ectopic pregnancy.Which of the following findings will confirm ectopic pregnancy? ¢  a.progesterone assay -30ng/dl ¢  b.beta HCG of 1800 iu/ml ¢  c.beta HCG determination 0f 800 increasing to 1800 in 48 hrs ¢  d.arias stella reaction on dilatation and curettage

¢  8.G1P0

¢  9.Which

of the following is not a part of the spigielberg criteria for the diagnosis of Ovarian pregnancy? ¢  a.ipsilateral tube is intact and distinct from the ovary ¢  b.ectopic pregnancy occupies the ovary ¢  c.ectopic pregnancy is connected to the uterus by the infundibulopelvic ligament ¢  d.ovarian tissues can be identified histologically amid placental tissues

¢  10.Which

of the following patients will have the highest risk for development of H mole? ¢  a.22 y/o filipino ¢  b. thailander with previous partial mole ¢  c.american smoker ¢  d.G3P0 ,hypertensive

¢  11.Which

¢  12.Which

¢  13.G1P0

¢  14.A

of the following is found in partial mole? ¢  a.beta HCG of more than 100,000 ¢  b.uterus is larger than AOG ¢  c.mistaken as missed abortion ¢  d.P57Kip 2 negative

PU 15 weeks has an ultrasound result of snowstorm pattern on ultrasound.Beta HCG -120,000IU/ML.The BEST management for the patient is ¢  a.methotrexate ¢  b.dilatation and curettage ¢  c.suction curettage ¢  d.hysterotomy

6 weeks complains of vaginal spotting and abdominal pain.cervix tender on wriggling,uterus slightly enlarged,with tenderness on the left adnexa.TVS revealed a 2 cm mass on the left adnexa with ring of fire pattern and fetal heart beats.No free fluid seen in the culde sac.What is the best management for the patient? ¢  a.medical with methotrexate ¢  b.laparoscopic salpingostomy ¢  c.laparoscopic salpingotomy ¢  d.laparotomy,salpingectomy

of the following is NOT is gestational trophoblastic neoplasia? ¢  a.complete mole ¢  b.placental site trophoblastic tumor ¢  c.invasive mole ¢  d.choriocarcinoma

24y/o G3P2 has a complete mole at 14 weeks AOG with bilateral asymptomatic theca lutein cyst measuring 8 cms.The management for the ovarian cyst is ¢  a.expectant ¢  b.oophorocystectomy ¢  c.aspiration ¢  d.oophorectomy

¢  15.Which

of the following is TRUE of gestational choriocarcinoma? ¢  a.Majority follows an H mole ¢  b.dilated villi microscopically ¢  c.lesions in the vagina must be biopsied ¢  d.lungs is most common site of metastasis

¢  16.36y/o

G7P5 underwent dilatation and curettage for a septic abortion .She complained of severe abdominal pain 12 hours after.Abdominal examination,with muscle guarding,with rebound tenderness.What complication do you consider in this case? ¢  a.uterine rupture ¢  b.uterine perforation ¢  c.endometritis ¢  d.embolism

¢  17.A

¢  18.40y/o

G4P3 16 weeks AOG came back with an ultrasound result showing intrauterine cystic masses exhibiting snowstorm pattern.Beta HCG-120,000 IU/ml.What is the best management for this patient ? ¢  a.methotrexate ¢  b.dilatation and curettage ¢  c.suction currettage ¢  d.hysterectomy

¢  19.What

¢  20.What

PLACENTA PREVIA

CLASSIFICATION

¢  The

¢  1.PLACENTA

placenta goes before the fetus in the birth canal

¢  Trophotropism-with

greater upper uterine flow the placental growth is more towards the fundus

¢  Placental

migration is a misnomer can wait till 34 weeks for trophotropism to occur

¢  We

is the most common predisposing factor for a tubal pregnancy? ¢  a.previous infection ¢  b.tubal surgery ¢  c.endometriosis ¢  d.smoking

PREVIA-internal os is partially or totally covered by the placenta

¢  2.LOW

LYING-implantation in the lower uterine segment but the edge does not reach the internal os and remains outside a 2 cm wide perimeter

¢  MARGINAL

PREVIA-placenta at the edge but does not overlie it

¢  VASA

PREVIA-fetal vessels course through the membranes and present at the cervical os

G3P3 postpartum for 3 months complains of vaginal bleeding since her last delivery.On examination ,a bluish mass which bleeds to touch is noted on the anterior vaginal wall and uterus is enlarged to 3 months size.What test will you request to arrive at a diagnosis? ¢  a.beta HCG ¢  b.transvaginal ultrasound ¢  c.biopsy of the mass ¢  d.dilatation and currettage

will differentiate a twin molar pregancy from a partial mole? ¢  a.beta HCG level ¢  b.p57kip 2 ¢  c.fetal karyotyping ¢  d.ultrasound findings

RISK FACTORS ¢  1.Multifetal

gestation age ¢  3.Multiparity ¢  4.Prior Cesarean section-MOST COMMON ¢  5.Cigarette smoking ¢  6.Elevated Prenatal Screening MSAFP levels ¢  2.Maternal

CLINICAL FEATURES

DIAGNOSIS

MANAGEMENT

¢  1.Painless

¢  1.Sonography-transabdominal,transvaginal

¢  PRETERM/NO

bleeding-MOST characteristic event

¢  2.abnormal

(more accurate) and transperineal

adherence of the placenta

PERSISTENT ACTIVE BLEEDING ¢  -observation,tocolytic,steroids

¢  2.MRI-expensive

¢  Placenta

accreta -ADHERENT Increta -INVADES ¢  Placenta percreta-PENETRATES

¢  3.IE

under double set up-done in the operating room and with preparation for emergency CS

¢  Placenta

PLACENTA ACCRETA

CLINICAL PRESENTATION AND DIAGNOSIS

¢  Partial

¢  Hemorrhage

or total absence of decidua basalis

TERM/NOT BLEEDING cesarean section(36-37 weeks) ¢  Low transverse(cutting through the placenta)) ¢  Vertical incision ¢  For anterior placenta previa with a previous CS scar-increase likelihood of placenta accreta ¢  -scheduled

MANAGEMENT

¢  -absent

Multidisciplinary approach Prepare blood for transfusion Tertiary Hospital Elective CS after 36 weeks Classical CS with Hysterectomy Do not attempt to remove the placenta Other Option:Leave the placenta and give methotrexate (not prepared for hysterectomy/ desirous of another pregnancy)

ABRUPTIO PLACENTA

RISK FACTORS

CLINICAL MANIFESTATION

¢  Premature

¢ 

PRIOR ABRUPTIO– most common Increasing age and Parity ¢  Hypertension ¢  Chorioamnionitis ¢  PPROM ¢  Multifetal pregnancy ¢  Low birthweight ¢  Hydramnios ¢  Cigarette smoking ¢  Thrombophilias ¢  Cocaine ¢  Uterine leiomyoma

¢  Sudden

¢ 

¢  Vaginal

development of the fibrinoid layer or nitabuchs layer

bleeding

together with previa or third stage

¢  NEAR

¢  Imperfect

¢  RISK

FACTORS;placenta previa and previous cesarean section,widespread use of MSAFP and HCGscreening for aneuploidies

placenta

¢  Total

separation of the normally implanted

or partial

¢  External

hemorrhage or concealed (more DIC)

¢  SONOGRAPHY/COLOR

MAPPING sonoluscent space between the placenta and decidua ¢  uterine serosa –bladder wall interface with retroplacental vessels is 50 ng/ml ¢  Less

than 24 WEEKS-patient counselling,expectant,Grp.B strep prophylaxis,antimicrobial

abortion smoking,inadequate maternal weight gain,illicit drug ¢  Genetic factor ¢  Birth defects ¢  Periodontal disease ¢  Interval between pregnancies ¢  Prior preterm birth-MOST COMMON ¢  Infection-bacterial vaginosis ¢  Cigarette

CORTICOSTEROID THERAPY

POSTTERM PREGNANCY

COMPLICATIONS

¢  Given

¢  Also

¢  1.Oligohydramnios

¢  Can

at 24 to 32 weeks also be given at 32-34 weeks

called prolonged pregnancy –extended pregnancy

¢  2.Macrosomia ¢  PATHOPHYSIOLOGY:

¢  TYPES; ¢  Betamethasone-12

¢  1.Postmaturity

¢  Dexamethasone-6

¢  2.placental

mg every 24 hours for 2 doses mg every 12 hours for 4 doses

syndrome dysfunction ¢  3.fetal distress and oligohydramnios ¢  4.fetal growth restriction

¢  3.Medical

and obstetrical complications

MANAGEMENT ¢  ¢ 

¢ 

¢ 

¢ 

1.UNFAVORABLE CERVIX a.cervical ripening agents-PGE2 and membrane stripping or sweeping 42 weeks with fetal compromise and oligohydramnios -INDUCE 42 weeks ,no complications ,favorable cervix-INDUCE

GROWTH RESTRICTION

RISK FACTORS AND ETIOLOGIES

¢  1.Symmetrical-proportionately

¢  Small

small,early insult,decrease in cell number abdominal size. ¢  Causes:chemical exposure,viral infection,cellular maldevelopment ¢  2.Asymmetrical-disproportionately

lagging abdominal growth,late insult such as placental insufficiency(hypertension)

42 weeks ,no complications,unfavorable cervixFETAL SURVEILLANCE WITH AF ASSESSMENT

mothers maternal weight gain ¢  Poor nutrition ¢  Social deprivation ¢  Drugs and teratogen ¢  Infection ¢  Fetal malformations ¢  Genetic abnormalities and multifetal pregnancy ¢  Placenta and cord abnormalites ¢  Poor

DIAGNOSIS

MANAGEMENT

MACROSOMIA

¢  Uterine

¢  >34

¢  Newborn

Fundal height-18-30 weeks the fundic height is almost equal as the age of gestation

¢  Ultrasound ¢  Amniotic ¢  Doppler

fluid measurement

velocimetry

WEEKS-doppler/NST/BPS weekly and fetal growth every 3-4 weeks

¢  DELIVER

IF WITH absent or reversed end diastolic flow,oligohydramnios,non reassuring fetal status,or with obstetrical indication to terminate,no growth in the biometry

¢  >24

but less than 34 WEEKS-SAME as above except you will give steroids

MANAGEMENT

QUESTIONS:

¢  DIABETIC

¢  1.Which

with EFW-4250 grams cesarean section ¢  Non diabetic.->5000 grams ¢  Prophylactic Induction of labor-proposed to reduce cesarean delivery and shoulder dystocia HOWEVER,studies showed it increases risk of CS without improved neonatal outcomes

of the following will put the patient at the highest risk for the development of Preterm Labor? ¢  a.multiparity ¢  b.smoking ¢  c.prior preterm birth ¢  d.infection

who weighed >4500 grams FACTORS: ¢  1.Obesity ¢  2.Diabetes-gestational ,type2-MOST COMMON ¢  3.postterm gestation ¢  4.multiparity ¢  5.large size parents ¢  6.advancing maternal age ¢  7.previous macrosomic infant ¢  8.racial and ethnic factors ¢  RISK

¢  2.G3P2(0-2-0-0)

PU 32 weeks has watery vaginal discharge.Nitrazine paper test –positive(yellow to blue).What is the management? ¢  a.tocolytic ¢  b.steroid andtocolytic ¢  c.expectant,steroid and ampicillin ¢  d.steroid,tocolytic.ampicillin

¢  3.Which

of the following findings is indicative of preterm labor? ¢  a.uterine contractions with closed cervix ¢  b.cervical length of 20mm ¢  c.fibronectin -20 ng/ml ¢  d.hypogastric pain

¢  4.30

y/o G4P3 PU 30 weeks was seen.Pregnancy test was positive at 4 weeks AOG.Fundic height -24 cms FHT-140/min.What is the assessment of this pregnancy ? ¢  a.normal pregnancy ¢  b.inaaccurate aging ¢  c.intrauterine growth restriction ¢  d. large for date pregnancy

¢  5.A

¢  6.What

¢  7.A

¢  8.A

is the most common risk factor for the development of fetal macrosomia? ¢  a.obesity ¢  b.diabetes ¢  c.multiparity ¢  d.nutrition

G4P3 PU 32 weeks has a fundic height of 24 cms.Biometry revealed a BPD /femur length compatible with 30 weeks and an abdominal circumference compatible with 24 weeks AOG .Which of the following is the cause of this condition? ¢  a.genetic ¢  b.chemical exposure ¢  c.hypertension ¢  d.viral infection

35 y/o G3P2 PU 41 weeks has an ultrasound findings of BPS-6/8 with AFI 4 cms.Cervix is closed and 1.5 cms long cephalic station 0.What is the best management ? ¢  a.hydrate patient ¢  b.CST and induce if negative ¢  c.close fetal surveillance ¢  d.cesarean delivery

G1P0 PU 38 weeks has a fundic height of 39 cms.Estimated fetal weight by ultrasound is 4250 grams.Her 75 gms OGTT revealed FBS -105 mg/ dl and 2nd hr -160mg/dl.What is the management? ¢  a.wait for spontaneous labor ¢  b.induce labor with prostaglandin ¢  c.Wait for 39 weeks and induce with oxytocin ¢  d.elective CS at 39 weeks

DYSTOCIA ¢  9.G2P1

PU 34 weeks,cephalic has a fundic height of 26 cms.Doppler velocimetry is requested every week to monitor the fetus.Which of the following findings will indicate severe fetal compromise? ¢  a.increase resistance index ¢  b.diastolic notching ¢  c.absent end diastolic flow ¢  d.reversed end diastolic flow

¢  10.G1P0

PU 42 weeks has an AFI-2cms.cervix closed ,uneffaced but soft.Which of the following is the best to induce labor in this patient?? ¢  a.membrane sweeping ¢  b.oxytocin ¢  c.prostaglandin ¢  d.primrose oil

¢  COMMON

CLINICAL FINDINGS WITH INEFFECTIVE LABOR ¢  1.Inadequate cervical dilatation and descent Protracted-slow progress Arrested-no progress Inadequate expulsive efforts 2.Fetopelvic disproportion Fetal size,pelvis,malpresentation or position 3.Ruptured membranes without labor

UTERINE DYSFUNCTION

PROLONGATION DISORDER

ACTIVE PHASE DISORDERS

¢  HYPOTONIC-

¢  Prolonged

¢ 

¢  >20

¢ 

no basal hypertonus,uterine contractions have normal gradient pattern(synchronous) but insufficient to dilate the cervix ¢  Treatment: Oxytocin ¢  HYPERTONIC –incoordinate,elevated basal tone,distorted pressure gradient ¢  Treatment: sedation

SECOND STAGE DISORDERS ¢  PROLONGED ¢  Nulliparas-

SECOND STAGE OF LABOR

2 hours /if with epidural-3 hours

¢  Multiparas-1

hour/if with epidural- 2hours

Latent phase hours-Nullipara ¢  >14 hrs –Multipara ¢  Treatment:Bed rest ¢  Exceptional Treatment:Oxytocin or cesarean delivery for urgent problems

PROTRACTION DISORDERS:Treatment-Expectant Protracted active phase-1 M ¢  Arrest in dilatation->2 hrs in Nulli and multi ¢  Arrest in descent ->1 hr in Nulli and Multi ¢  Failure in descent –No descent in deceleration phase ¢  ¢ 

REVISED GUIDELINES ON ARREST DISORDERS

CAUSES OF UTERINE DYSFUNCTION

¢  Arrest

¢  Epidural

of cervical dilatation- 6cms with membrane rupture for 4 hours with 200 montevideo units uterine contractions ¢  Or 6 hrs if with no adequate contractions ¢  Prolonged

second stage- 3 hrs for nulliparas without epidural and 4 hrs if with epidural

anesthesia

¢  Chorioamnionitis ¢  Maternal

position during labor-ambulation positionupright,sitting,kneeling,squatting,30 degree elevation ¢  Water immersion ¢  Birthing

PRECIPITOUS LABOR AND DELIVERY

PELVIC CONTRACTION

CONTRACTED MIDPELVIS

¢  Delivery

¢  1.INLET

¢  Causes

¢  Obstetric

¢  Interspinous

of the fetus in less than 3 hours

¢  Cervical

dilatation 5cms/hr in nullipara and 10 cms /hr in multipara

¢  Maternal

and fetal effects:amniotic fluid embolism,uterine atony,abruptio,meconium,low apgar score

¢  TREATMENT;Discontinue

oxytocin,analgesic,tocolytic,general anesthesia

cojugate less than 10 cms,diagonal conjugate less than 11.5 cms and GTI less than 12 cms ¢  Associated with asynclitism ,early rupture of membranes and descent doesn’t occur until after labor onset,less efficient uterine contractions,malpresentations,cord prolapse

transverse arrest of the head diameter is less than 8 cms ¢  Sum of the interpinous(10.5) plus posterior sagittal diameter (5) is less than 13.5 cms ¢  CLINICALLY:prominent spines,convergent side walls,narrow sacrosciatic notch

CONTRACTED OUTLET

MUELLER HILLIS MANEUVER

¢  Interischial

¢  Fetal

¢  Often

tuberous diameter is 8 cms or less associated with midpelvic contraction

FACE PRESENTATION The occiput is the longer end of the head lever. The chin is directly posterior. Vaginal delivery is impossible unless the chin rotates anteriorly

FACE PRESENTATION

brow and suboccipital region are grasped to the abdominal wall with the fingers and firm pressure is directed downward along the axis of the inlet

—  —  — 

Descent - same factors as in cephalic presentations internal rotation - the objective is to bring the chin under the symphysis pubis - same factors as in vertex presentations flexion extension external rotation - results from the relation of the fetal body to the deflected head

is hyperextended , occiput is in contact with the fetal back and the chin (mentum) is presenting ¢  fetal face may present with the chin (mentum) anteriorly or posteriorly, relative to the maternal symphysis pubis

FACE PRESENTATION

Etiology ¢  Prematurity ¢  Marked enlargement of the neck or coils of cord about the neck may cause extension ¢  Anencephalic fetuses ¢  Contracted pelvis ¢  Very large fetus ¢  Multiparous women ¢  Hydramnios

Diagnosis

FACE PRESENTATION

BROW PRESENTATION

Management — 

¢  head

FACE PRESENTATION

Mechanism of Labor — 

FACE PRESENTATION

¢  Vaginal

Delivery (Mentum Anterior) delivery (Mentum Posterior)

¢  Cesarean

¢  Vaginal — 

examination

palpation of the distinctive facial features of the mouth and nose, the malar bones, and particularly the orbital ridges (differentiate it from breech)

¢  Radiographic

¢  Rarest

examination

presentation because it often converts to face or occiput presentation ¢  fetal head between the orbital ridge and anterior fontanel presents at the pelvic inlet ¢  fetal head occupies a position midway between full flexion (occiput) and extension (mentum or face)

BROW PRESENTATION

BROW PRESENTATION

Diagnosis

¢  Vaginal

Mechanism of Labor ¢  very small fetus and a large pelvis - labor is generally easy ¢  larger fetus - usually difficult ¢  Persistent Brow – vaginal delivery is difficult and management is same as face

TRANSVERSE LIE

TRANSVERSE LIE

Etiology ¢  Abdominal wall relaxation from high parity. ¢  Preterm fetus. ¢  Placenta previa. ¢  Abnormal uterine anatomy. ¢  Excessive amnionic fluid. ¢  Contracted pelvis.

Diagnosis ¢  Abdominal examination

¢  Abdominal

palpation - when both the occiput and chin can be palpated easily examination – palpation of the frontal sutures, large anterior fontanel, orbital ridges, eyes, and root of the nose

TRANSVERSE LIE ¢  the

long axis of the fetus is approximately perpendicular to that of the mother ¢  Long axis forms an acute angle – Oblique Lie (unstable) ¢  referred to as shoulder or acromnion presentation ¢  the shoulder is usually on the pelvic inlet, with the head lying on one iliac fossa and the breech in another

TRANSVERSE LIE

— 

—  — 

no fetal pole is detected in the fundus, ballottable head is found in one iliac fossa and the breech in the other (anterior) - a hard resistance plane extends across the front of the abdomen (posterior)- irregular nodulations representing the small parts are felt through the abdominal wall.

Palpation in transverse lie, right acromidorsoanterior position. A. First maneuver. B. Second maneuver. C. Third maneuver. D. Fourth maneuver.

TRANSVERSE LIE

TRANSVERSE LIE

Diagnosis

Management ¢  Indication for cesarean delivery ¢  vertical incision is recommended especially for dorsoanterior presentation(Back down)

¢  Vaginal —  — 

examination

early stages of labor: the side of the thorax or the "gridiron" feel of the ribs Advanced labor: the scapula and clavicle are palpated

CONDUPLICAT O CORPORE Neglected shoulder presentation. A thick muscular band forming a pathological retraction ring

has developed just above the thin lower uterine segment. The force generated during a uterine contraction is directed centripetally at and above the level of the pathological retraction ring. This serves to stretch further and possibly to rupture the thin lower segment below the retraction ring. (P.R.R. = pathological retraction ring.)

COMPOUND PRESENTATION

COMPOUND PRESENTATION Causes

¢  an

extremity prolapses alongside the presenting part or with both presenting in the pelvis simultaneously

¢  conditions

that prevent complete occlusion of the pelvic inlet by the fetal head, including preterm birth

COMPOUND PRESENTATION Prognosis and Management ¢  preterm delivery, prolapsed cord, and traumatic obstetrical procedures ¢  Prolapsed

arm alongside the head ascertain whether the arm retracts out of the way with descent of the presenting part, if it fails to retract and if it appears to prevent descent of the head, the prolapsed arm should be pushed gently upward and the head simultaneously downward by fundal pressure

PERSISTENT OCCIPUT POSTERIOR POSITION

RISK FACTORS FOR PERSISTENT OCCIPUT POSTERIOR (POP)

MORBIDITY ASSOCIATED WITH POP ¢  Prolonged ¢  Increased

¢  Transverse

narrowing of the midpelvis

¢  Usually

undergo spontaneous anterior rotation followed by uncomplicated delivery

DELIVERY OF PERSISTENT OCCIPUT POSTERIOR Spontaneous Vaginal Delivery

1.  — 

Roomy pelvic outlet or relaxed perineum

Manual rotation to occiput anterior and spontaneous delivery

2.  — 

Resistant vaginal outlet or form perineum

Forceps or Vacuum delivery

3.  —  — 

Ineffective expulsive efforts Must meet criteria for forceps or vacuum delivery

Cesarean section

4.  —  — 

Elongation of fetal head (molding/caput) Head not engaged

¢  Epidural

analgesic ¢  Nulliparity ¢  Greater fetal weight ¢  Prior Occiput posterior position delivery

PERSISTENT OCCIPUT TRANSVERSE POSITION occiput tends to rotate to anterior position in the absence of a pelvic architecture abnormality or asynclitism.May use (Kielland forceps) ¢  Spontaneous delivery or delivery with outlet forceps.(simpsons) ¢ 

second stage of labor CS delivery and operative vaginal

delivery ¢  Increased

blood loss (vaginal delivery) order vaginal lacerations (3rd and 4th degree lacerations)

¢  Higher

SHOULDER DYSTOCIA ¢  Head —  — 

to body delivery time

Normal birth – 24 seconds Shoulder dystocia - > 60 seconds

¢  Fetal

shoulder become wedged behind symphysis pubis and fail to deliver ¢  EMERGENCY – because the umbilical cord is compressed within the birth canal.

PREDICTORS FOR SHOULDER DYSTOCIA

SHOULDER DYSTOCIA

SHOULDER DYSTOCIA Management

Maternal Consequences

Increasing fetal weight risk factors:

1. 

Obesity Multiparity —  Diabetes Mellitus and Gestational Diabetes Mellitus —  Post term pregnancy 75 % shoulder dystocia cases Birthweight > 4000 grams — 

¢  Postpartum

hemorrhage - usually from uterine atony, vaginal and cervical lacerations

Fetal Consequences ¢  Fetal morbidity and mortality (Neuromusculoskeletal injuries) ¢  Brachial Plexus Injury ¢  Clavicular fracture/Humeral fracture/Rib Fracture ¢  Hypoxia

— 

Intrapartum Factors:

2.  —  —  — 

¢  Reduction

in the interval of time from delivery of the head to delivery of the body ¢  initial gentle traction, assisted by maternal expulsive efforts ¢  Large episiotomy ¢  Adequate analgesic

Prolonged second stage Operative vaginal delivery Prior shoulder dystocia

SHOULDER DYSTOCIA

TECHNIQUES TO FREE THE ANTERIOR SHOULDER FROM ITS IMPACTED POSITION BENEATH THE SYMPHYSIS PUBIS: ¢  McRoberts

SHOULDER DYSTOCIA

TECHNIQUES TO FREE THE ANTERIOR SHOULDER FROM ITS ¢  IMPACTED Moderate POSITION suprapubic pressure BENEATH THE SYMPHYSIS PUBIS: —  — 

— 

downward traction is applied to the fetal head. Pressure is applied with the heel of the hand on the anterior shoulder

—  — 

Shoulder dystocia with impacted anterior shoulder of the fetus.

SHOULDER DYSTOCIA The McRoberts maneuver. The maneuver consists of removing the legs from the stirrups and sharply flexing the thighs up onto the abdomen, as shown by the horizontal arrow. The assistant is also providing suprapubic pressure simultaneously (vertical arrow).  

A.The operator's hand is introduced into the vagina along the fetal posterior humerus, which is splinted as the arm is swept across the chest, keeping the arm flexed at the elbow. B. The fetal hand is grasped and the arm extended along the side of the face. C. The posterior arm is delivered from the vagina

maneuver

consists of removing the legs from the stirrups and sharply flexing them up onto the abdomen rotation of the symphysis pubis toward the maternal head, and a decrease in the angle of pelvic inclination pelvic rotation cephalad tends to free the impacted anterior shoulder

SHOULDER DYSTOCIA

TECHNIQUES TO FREE THE ANTERIOR SHOULDER FROM ITS IMPACTED POSITION BENEATH THE SYMPHYSIS PUBIS: ¢  Woods

corkscrew maneuver

The hand is placed behind the posterior shoulder of the fetus and progressively rotating the posterior shoulder 180 degrees in a corkscrew fashion so the impacted anterior shoulder could be released

SHOULDER DYSTOCIA ¢  Rubin’s — 

¢ 

Deliberate fracture of the clavicle

¢ 

Hibbard Maneuver

maneuver

Applying pressure on the posterior aspect of the anterior shoulder towards the chest

— 

— 

¢ 

Cleidotomy — 

¢ 

6.  7. 

Delivery of posterior arm Woodscrew Rubin’s manever

IF IT FAILS… 8.  9.  10. 

Cleidotomy Zavanelli Symphysiotomy

pressure is applied to the fetal jaw and neck in the direction of the maternal rectum, with strong fundal pressure applied by an assistant as the anterior shoulder is freed

Zavanelli maneuver

— 

5. 

pressing the anterior clavicle against the ramus of the pubis to free the shoulder impaction

¢ 

— 

IF THE ABOVE MANEUVERS FAIL:

SHOULDER DYSTOCIA DRILL

SHOULDER DYSTOCIA

cephalic replacement into the pelvis and then cesarean delivery. cutting the clavicle with scissors or other sharp instruments usually used for a dead fetus

Symphysiotomy

COMPLICATIONS WITH DYSTOCIA

1. Call for help—mobilize assistants, an anesthesiologist, and a pediatrician. Initially, a gentle attempt at traction is made. Drain the bladder if it is distended. 2. A generous episiotomy (mediolateral or episioproctotomy) may afford room posteriorly. 3. Suprapubic pressure is used initially by most practitioners because it has the advantage of simplicity. Only one assistant is needed to provide suprapubic pressure while normal downward traction is applied to the fetal head. 4. The McRoberts maneuver requires two assistants.

TYPES OF BREECH PRESENTATION ¢  FRANK-

MATERNAL ¢  Uterine

rupture ¢  Pathological retraction ring ¢  Fistula formation ¢  Pelvic floor injury ¢  Infection ¢  Postpartum hemorrhage

thighs are flexed and legs are extended

PERINATAL ¢  Fetal

sepsis ¢  Caput succedaneum ¢  Molding ¢  Nerve injury/fractures ¢  Cephalhematoma

¢  COMPLETE-

thighs and legs are both flexed

¢  INCOMPLETE(FOOTLING)

one or both thighs or legs is /are extended and maybe prolapsed into the vagina

EXTERNAL CEPHALIC VERSION

CONTRAINDICATIONS TO ECV

¢  The

¢  Any

fetus is turned in utero by manipulation of the maternal abdomen from a non cephalic to cephalic ¢  Should be performed at more than 36 weeks ¢  PREREQUISITES TO ECV — 

Singleton,AOG- more than or equal to 37 weeks,no contraindications to labor,fetal well being established prior to procedure, amniotic fluid and position of fetus are known,facilities for immediate delivery

—  —  — 

contraindications to labor

placenta previa, non reassuring fetal heart rate, IUGR

¢  Congenital ¢  HIV

anomalies (hydrocephalus) patients

TECHNIQUE OF EXTERNAL CEPHALIC VERSION

TECHNIQUE

¢  Must

be performed in a facility equipped to perform emergency CS ¢  Sonographic evaluation to determine presentation,amniotic fluid,exclude fetal anomalies,location of placenta ¢  External monitoring of fetal heart rate reactivity ¢  Give anti D immunoglobulin for Rh negative patients

¢  FORWARD

PLANNED VAGINAL BREECH DELIVERY

TYPES OF VAGINAL BREECH DELIVERY

TECHNIQUE FOR VAGINAL BREECH DELIVERY

¢  Frank

¢  SPONTANEOUS

¢  Ensure

or complete breech at 36 weeks or more with estimated fetal weight of 2500 grams to 4000 gms ¢  Frank or complete breech at 31-35 weeks when estimated fetal weight is 1500-2500 grams BUT THE WOMANS WISHES IN COLLABORATION WITH THE ATTENDING PHYSICIANS JUDGEMENT SHOULD DETERMINE WHICH DELIVERY METHOD IS MOST APPROPRIATE

PINARDS MANEUVER ¢  Insert

two fingers along one extremity to the knee ¢  Abduction of the thigh away from the midline ¢  Apply pressure on the popliteal fossa to flex the legs and delivery of the foot

ROLL-each hand grasps one of the fetal poles and the buttocks are elevated from the maternal pelvis and displaced laterally.The buttocks are gently guided toward the fundus while the head is directed toward the pelvis ¢  BACKWARD FLIP

BREECH DELIVERY- the fetus is expelled entirely without traction or manipulation other than support of the newborn ¢  PARTIAL BREECH DELIVERY-fetus is delivered spontaneously up to the umbilicus and the remainder of the body is extracted with operators traction and maneuvers ¢  TOTAL BREECH DELIVERY- the entire body of the fetus is extracted by the obstetrician

adequate analgesia expulsion up to the navel with maternal pushing “DON’T PULL THE BREECH” ¢  Facilitate rotation to sacrum anterior ¢  Episiotomy when buttocks and anus are crowning ¢  If legs are not delivered spontaneously do PINARDS MANEUVER ¢  Spontaneous

TECHNIQUE OF VAGINAL BREECH DELIVERY ¢  Support

the baby around the hips and ask the woman to push until scapulae are visible

¢  DON’T

HOLD THE FLANK OR ABDOMEN,DO NOT PULL THE BREECH

¢  Maintain

flexion of the fetal head by keeping the body below the horizontal

DELIVERY OF THE ARMS

DELIVERY OF THE FETAL HEAD

¢  Rotate

¢  MAURICEAU-SMELLIE

the body to facilitate delivery of the arms MANEUVER ¢  When the anterior shoulder and arm appear at the vulva two fingers are applied on the antecubital fossa to flex the arm and sweep it across the chest as if a cat is washing his face

–VEIT MANEUVER

¢  LOVESET

¢  The

body of the fetus is then rotated in the reverse direction to deliver the other shoulder and arm

— 

Maintain the head in flexion

— 

Place the attendants two fingers on the chin and malar eminences

— 

The assistant help by providing suprapubic pressure as traction is applied by the primary health care provider

HEAD ENTRAPMENT

DELIVERY OF THE FETAL HEAD USE OF PIPERS FORCEPS The fetal body is elevated using warm towel

¢  Duhrssens — 

Left blade is applied to the aftercoming head

¢  Zavanelli

The right blade is applied with the body still elevated

Incision-2 , 10 o clock position

ADD 6 OCLOCK POSITION IF NOT SUCCESSFUL

maneuver-cephalic replacement

¢  Symphysiotomy

Embryogenesis of the Reproductive Tract ¢  UTERUS

ABNORMALITIES OF THE REPRODUCTIVE TRACT

DEPARTMENT OF OBSTETRICS AND GYNECOLOGY

FEU – NRMF INSTITUTE OF MEDICINE

¢ 

formed by the union of the 2 mullerian ducts

—  — 

Upper third of vagina-mullerian duct Lower vagina –urogenital sinus

VULVAR ABNORMALITIES Imperforate hymen ¢  fusion between the sinovaginal bulbs with the urogenital sinus. ¢  primary amenorrhea and ¢  Cyclic pelvic pain ¢  Bulging mass at the introitus ¢  Hematocolpos/hematometra ¢  Treatment:cruciate incision of the hymen

Vaginal Abnormalities ¢  — 

VAGINAL ABNORMALITIES

Vaginal agenesis Failure of dissolution of the cell cord between the urogenital sinus and mullerian tuburcle

—  — 

Transverse septate vagina

¢  — 

— 

¢  This

anomaly is distinguished from bicornuate and septate uteri by the presence of complete nonfusion of the cervix and hemiuterine cavity. ¢  Complications may include - preterm delivery (20%) - fetal growth restriction (10%) - breech presentation (43%) - cesarean delivery rate (82%)

Results from faulty canalization of the fused mullerian anlage

UNICORNUATE UTERUS (CLASS II) — 

UTERINE DIDELPHYS (CLASS III)

Partial or complete Associated with Rokitansky-Kuster-Hauser syndrome or androgen insensitivity

Longitudinal septate vagina

¢ 

CLASS II

UTERINE MALFORMATIONS

Vaginal atresia

¢ 

—  —  — 

Discovered by: routine pelvic examination. cesarean delivery or during manual exploration of the uterine cavity after delivery.

Obstetrical Significance —  Results in miscarriage, ectopic pregnancy, rudimentary horn pregnancy, preterm delivery, fetal growth restriction, abnormal fetal lie, uterine dysfunction, or uterine rupture.

CLASS III

increased incidence of infertility, endometriosis, and dysmenorrhea. Pregnancy in the normal-sized hemiuterus : - preterm delivery - fetal growth restriction - breech presentation - dysfunctional labor - cesarean delivery

CLASS IV

CLASS V

BICORNUATE AND SEPTATE UTERI (CLASSES IV AND V)

ACQUIRED REPRODUCTIVE TRACT ABNORMALITIES

ACQUIRED REPRODUCTIVE TRACT ABNORMALITIES

Vulvar Abnormalities

¢  Marked

increase in miscarriages ¢  preterm delivery, abnormal fetal lie, and cesarean delivery.

¢ 

¢ 

Uterine Abnormalities Anteflexion ¢  Due to diastasis recti and a pendulous abdomen ¢  Management: abdominal binder.

Bartholin Gland Lesions —  Needs no treatment during pregnancy. —  If large enough to cause difficulty at delivery, - needle aspiration —  Abscess - broad-spectrum antimicrobials and drainage

Retroflexion ¢  uterus remains incarcerated in the hollow of the sacrum ¢  abdominal discomfort and inability to void normally. ¢  Management:Knee chest position

Condyloma Acuminata Vaginal delivery unless so extensive that vaginal delivery may be prohibited.

— 

 

UTERINE ABNORMALITIES

Uterine Prolapse —  — 

Management: pessary in early pregnancy

Cystocele and Rectocele — 

— 

UTERINE ABNORMALITIES

Leiomyoma Degeneration during pregnancy: red or carneous degeneration —  Management during pregnany: myomectomy NOT DONE during pregnancy.Pain reliever given for pain — 

*** Two factors for morbidity are: myoma SIZE and LOCATION

SECOND TRIMESTER-smaller myomas (2 to 6 cm) remained unchanged or increased in size, whereas larger myomas became smaller TRIMESTER- remained unchanged or decreased, (estrogen receptor downregulation).

   

  

Effects of Myoma in Pregnancy  —  preterm labor —  placental abruption —  malpresentation —  obstructed labor —  cesarean delivery —  postpartum hemorrhage

EFFECTS OF PREGNANCY ON MYOMA: FIRST TRIMESTER-remained unchanged or increased in size—due to estrogen.

¢  THIRD

Cystocele or a rectocele can block normal fetal descent Management:Catheterization

UTERINE ABNORMALITIES - MYOMA

UTERINE ABNORMALITIES - MYOMA

UTERINE ABNORMALITIES - MYOMA Management of Myomas during Pregnancy ¢  expectant management ¢  Resection of myomas is generally contraindicated.

OVARIAN ABNORMALITIES ¢  Mostly

cystic. by ultrasound or at cesarean delivery. ¢  corpus luteal cysts,/ luteomas, which may be virilizing. ¢  Ovarian hyperstimulation syndrome,caused by ovulation-induction ¢  Second to malignancy, the most serious complications are torsion or hemorrhage. ¢  Diagnosed

MANAGEMENT ¢  Resection

of all cysts is recommended in:

>10cm Suspected rupture or torsion ¢ Capable of obstructing labor ¢  ¢ 

¢  Cysts

5cm or less – left alone, most undergo resolution

OVARIAN ABNORMALITIES Recommendations:WHEN TO OPERATE —  All ovarian masses over 10 cm —  6 to 10 cm with signs of malignancy by ultrasound and MRI —  Give 17-OH-progesterone, 250 mg intramuscularly weekly until 10 weeks if corpus luteum was removed — 

Elective surgery at 16 to 20 weeks

QUESTIONS: ¢  1.G1P0

PU 38 weeks in labor was admitted.Uterine contractons occurred every 2 minutes 60 secs duration.Cervix 2 cms dilated 1 cm long.After 24 hours,cervix is still 3 cms dilated 0.5 cms long.What is the best management? ¢  a.oxytocin ¢  b.sedation ¢  c.amniotomy ¢  d.cesarean section

¢  2.G1P0

39 weeks AOG admitted at 5 cms cervical dilatation 0.5 cms long cephalic station -1.Uterine contractions-200 montevideo units.Amniotomy done revealed clear AF.cervix dilated to 6 cms after an hour,cephalic station-1.However after 3 hours cervix remained at 6 cms.,cephalic station -1.What is the diagnosis? ¢  a.prolonged active phase ¢  b.protracted active phase ¢  c.arrest in cervical dilatation ¢  d.failure descent

¢  3.Failure

in descent can be diagnosed if there is no descent during which phase of labor? ¢  a.latent ¢  b.acceleration ¢  c.active ¢  d.deceleration

¢  4.Precipitate

¢  5.Clinical

¢  6.What

¢  7.G1P0

pelvimetry findings of a nulliparous patient revealed a prominent ischial spines,convergent sidewalls,narrow sacrosciatic notch.Which pelvic plane is contracted? ¢  a.inlet ¢  b.midplane ¢  c.outlet

plane of the pelvis is tested by theMueller Hillis Maneuver ? ¢  a.inlet ¢  b.midplane ¢  c.outlet

delivery can be diagnosed in a nulliparous patient if cervical dilatation is more than___cms/hr ¢  a.2 ¢  b.3 ¢  c.4 ¢  d.5

38 weeks AOG has this leopolds findings: L2-back on the right,small parts on the left,L3-cephalic L4 –cephalic prominence on the right.On IE the mentum was directed at the sacrum.What is the manner of delivery? ¢  a.NSD ¢  b.forceps ¢  c.vacuum ¢  d.cesarean ¢  L1-breech

¢  8.A

multipara in labor has this IE findings.The frontal sutures,anterior fontanel,orbital ridges and root of the nose are palpated.What is the presentation? ¢  a.sincipital ¢  b.brow ¢  c.face ¢  d.vertex

¢  9.A

multipara was admitted in active labor.IE revealed a gridiron feel with back down position.What is the best management? ¢  a.external cephalic version ¢  b.internal podalic version ¢  c.low segment cesarean ¢  d.classical cesarean

¢  10.What

¢  11.In

shoulder dystocia ,the procedure of hyperflexing the legs towards the abdomen is called ¢  a.pinards ¢  b.rubins ¢  c.mc roberts ¢  d.zavanelli

¢  12.External

cephalic version to convert a breech presentation to cephalic is recommended at what weeks age of gestation? ¢  a.33 ¢  b.35 ¢  c.37 ¢  d.39

¢  13.In

¢  14.Which

¢  15.A

¢  16.Which

of the following structures is NOT derived from the mullerian duct? ¢  a.uterus ¢  b.hymen ¢  c.upper third of the vagina ¢  d.cervix

G1P0 PU 12 weeks has a 15 cms asymptomatic,ovarian cyst on the left adnexa.What is the management? ¢  a.expectant ¢  b.immediate exploration ¢  c.explore at 16-20 weeks ¢  d. explore after delivery

forceps is used to rotate a persistent occiput transverse to anterior position? ¢  a.simpsons ¢  b.kiellands ¢  c.pipers ¢  d.bartons

partial breech extraction,the procedure of lateral deflection of the thigh,pressing on the popliteal to flex the legs and deliver the foot is called ¢  a.loveset ¢  b.hibbard ¢  c.pinard ¢  d.zavanelli

of the following is NOT used to deliver an entrapped head in breech presentation? ¢  a.rubins maneuver ¢  b.mauriceau smellie veit maneuver ¢  c.suprapubic pressure ¢  d.durshsen incision

¢  17.If

there is no union of the mullerian duct ,the abnormality produced is ¢  a.unicornuate uterus ¢  b.bicornuate ¢  c.uterus didelphys ¢  d.septate uterus

¢  18.A

17 year old consulted because of primary amenorrhea and cyclic pelvic pain.On examination,bulging mass was noted at the introitus with no vaginal opening.What is the diagnosis? ¢  a.endometrial polyp ¢  b.prolapsed myoma ¢  c.imperforate hymen ¢  d.vaginal septum

¢  19.19y/o

G1P0 PU 34 weeks has painful myoma uteri for 1 week.What is the degeneration of the myoma ? ¢  a.hyaline ¢  b.carneous ¢  c.cystic ¢  d.sarcomatous

ETIOLOGY OF MULTIPLE FETUSES ¢  20.G3P2

PU 36 weeks came in fully dilated,frank breech presentation ,station + 3.The attending physician waited for the spontaneous expulsion of the breech up to the navel and assist the delivery with maneuvers from navel up to the head.What is the described type of extraction? ¢  a.Spontaneous breech delivery ¢  b.total breech extraction ¢  c.complete breech extraction ¢  d.partial breech extraction

MULTIFETAL GESTATION DEPARTMENT OF OBSTETRICS AND GYNECOLOGY FEU – NRMF INSTITUTE OF MEDICINE

CAUSES: Superfetation ¢  An interval longer than a menstrual cycle intervenes between fertilizations. ¢  Not

yet proven in humans  Superfecundation ¢  Fertilization of two ova within the same menstrual cycle but not as the same coitus, nor necessarily by sperm from the same male.

MONOZYGOTIC

- relatively constant - largely independent of race, heredity, age and parity —  assisted reproductive therapy

Fraternal Twin ¢  Fertilization of two separate ova ¢  “double – ovum”, or dizygotic   Identical Twins ¢  Twins arise from single fertilized ovum ¢  “single-ovum”, monozygotic ¢  Have increased incidence of discordant malformations

GENESIS OF DIZYGOTIC TWINNING DIZYGOTIC ¢  heredity ¢  increasing

maternal age parity ¢  nutritional factors ¢  pituitary gonadotropin ¢  infertility therapy ¢  assisted reproductive therapy ¢  increasing

Genesis of Monozygotic Twinning

MONOZYGOTI C

CONJOINED TWIN

DETERMINATION OF ZYGOSITY

Infant Sex and blood type ¢ Twins of the opposite sex are almost always dizygotic ¢ Infants of different blood types are dizygotic

DETERMINATION OF ZYGOSITY Ultrasound – the number of placenta (chorionicity) can give a clue on zygosity —  Can determine chorionicity as early as the first trimester —  Dichorionic: presence of two separate placentas and a thick – generally 2mm or greater – dividing membrane (“twin – peak” sign) —  Monochorionic: membrane generally less than 2mm in thickness and reveals only 2 layers. ( “T” sign)  Placental Examination —  Visual examination of the placenta and membranes —  Placenta should be carefully delivered to preserve the attachment of the amnion and chorion to the placenta

DETERMINATION OF ZYGOSITY Ultrasound – the number of placenta (chorionicity) can give a clue on zygosity —  Can determine chorionicity as early as the first trimester —  Dichorionic: presence of two separate placentas and a thick – generally 2mm or greater – dividing membrane (“twin – peak” sign) —  Monochorionic: membrane generally less than 2mm in thickness and reveals only 2 layers. ( “T” sign)  Placental Examination —  Visual examination of the placenta and membranes —  Placenta should be carefully delivered to preserve the attachment of the amnion and chorion to the placenta

DETERMINATION OF ZYGOSITY

DIAGNOSIS OF MULTIPLE FETUSES

DIAGNOSIS OF MULTIPLE FETUSES Ultrasonography  ¢  Separate gestational sacs ¢  Two fetal heads or two abdomens should be seen in the same plane

Ultrasound – the number of placenta (chorionicity) can give a clue on zygosity —  Can determine chorionicity as early as the first trimester —  Dichorionic: presence of two separate placentas and a thick – generally 2mm or greater – dividing membrane (“twin – peak” sign) —  Monochorionic: membrane generally less than 2mm in thickness and reveals only 2 layers. ( “T” sign)  Placental Examination —  Visual examination of the placenta and membranes —  Placenta should be carefully delivered to preserve the attachment of the amnion and chorion to the placenta

DIFFERENTIAL DIAGNOSIS OF ENLARGED FUNDIC HEIGHT —  Multiple fetuses —  Elevation of the uterus by a distended bladder —  Inaccurate menstrual history —  Hydramnios —  Hydatidiform mole —  Uterine myomas —  A closely attached adnexal mass —  Fetal macrosomia (late in pregnancy)

DIAGNOSIS OF MULTIPLE FETUSES

MATERNAL ADAPTATION

PREGNANCY OUTCOME

Radiologic Examination ¢  Not useful

¢ Nausea

¢ Abortion

Biochemical Test BETA HCG- higher than singleton pregnancy, but not definite

and vomiting (severe) blood volume -greater ¢ Cardiac output - 20% greater than singleton ¢ Blood loss via NSD 1000 ml ¢ Maternal

PREGNANCY OUTCOME

PREGNANCY OUTCOME

¢  Malformation

¢ Low

- Defects from twinning itself. conjoined twinning, acardiac anomaly, sirenomelia, neuraltube defects, and holoprosencephaly. - Defects from vascular interchange between monochorionic twins. microcephaly, hydranencephaly, intestinal atresia, aplasia cutis, or limb amputation. - Defects as a result of crowding.

birthweight ( more on monozygotic than dizygotic) ¢ Preterm birth

—  3x

> than singleton pregnancies : Dizygotic Risk: 18:1

—  Monozygotic

UNIQUE COMPLICATIONS

MONOAMNIONIC TWINS

MONOAMNIONIC TWINS

CONJOINED TWINS

¢ Management

-high fetal death from cord entanglement - 1 hour daily FHR monitoring beginning at 26-28 weeks - corticosteroid therapy- to promote fetal lung maturity - CS at 34 weeks

¢ High

risk for fetal death : - cord entanglement - congenital anomaly - preterm birth - twin to twin transfusion sydrome

~ SIAMESE TWINS

TYPES OF CONJOINED TWINS

TYPES OF CONJOINED TWINS

TWIN TO TWIN TRANSFUSION

TWIN TO TWIN TRANSFUSION SYNDROME

ACARDIAC TWIN

¢  DONOR

— 

Blood is transfused from a donor twin to its recipient sibling such that the donor becomes anemic and its growth may be restricted, whereas the —  Recipient becomes polycythemic and may develop circulatory overload manifest as hydrops. —  Donor twin - pale, recipient sibling – plethoric Fetal consequences: —  circulatory overload with heart failure —  Occlusive thrombosis is also much more likely to develop in this setting. —  Polycythemia may lead to severe hyperbilirubinemia and kernicterus — 

¢ 

TWIN- anemic and growth restricted pale looking,oligohydramnios

¢  RECIPIENT

TWIN-polycythemic,phletoric ¢  circulatory overload,heart failure due to

— 

— 

¢ 

hypervolemia,hyperbilirubinemia ¢ 

kernicterus,polyhydramnios

— 

Twin reversed-arterial-perfusion (TRAP) sequence is a rare (1 in 35,000 births) but serious complication of monochorionic, monozygotic multiple gestation. In the TRAP sequence, there is usually a normally formed donor twin who has features of heart failure as well as a recipient twin who lacks a heart (acardius) and various other structures. Caused in the embryo by a large artery-to-artery placental shunt, often also accompanied by a vein-tovein shunt. The perfusion pressure of the donor twin overpowers that in the recipient twin, who thus receives reverse blood flow from its twin sibling.

TRAP SEQUENCE(TWIN REVERSED ARTERIAL PERFUSION)

MANAGEMENT

DISCORDANT TWINS

¢  Large

¢  Without

treatment, the donor or "pump" twin has been reported to die in 50 to 75 percent ¢  Methods of in utero treatment of acardiac twinning: goal is interruption of the vascular communication between the donor and recipient twins.

¢ Size

DIAGNOSIS OF DISCORDANCY

MANAGEMENT

artery to artery placental shunt to vein shunt ¢  Single shared placenta,the donor twin overpowers the recipient twin.The recipient twin utilizes the ‘used ‘arterial blood which perfuses the lower body and disrupts growth of the upper body ¢  Vein

¢  DONOR-

Heart Failure/RECIPIENT-Lacks heart

PATHOLOGY ¢  MONOCHORIONIC

TWINS- placental vascular anastomoses ¢  DIZYGOTIC TWINS- different genetic growth potential

¢  ULTRASOUND

ASSESSMENT:ABDOMINAL CIRCUMFERENCE ¢  DISCORDANCY-WT(large twin)- Wt (smaller twin) divided by weight of large twin ¢  DISCORDANCY

OF 25 to 30 % predicts adverse

outcome

DIET

ANTEPARTUM SURVEILLANCE ¢  Assessment

¢  37-54

lb weight gain ¢  INCREASED CALORIC INTAKE-40-45 KCAL/ DAY ¢  IRON- 60 to 100 mg/day of iroN

of amnionic fluid volumeOLIGOHYDRAMNIOS

¢  The

nonstress test or biophysical profile – FETAL WELL BEING

¢  Doppler

IUGR

evaluation of vascular resistance -

inequality of twin fetuses ¢ As the weight difference within a twin pair increases, perinatal mortality increases proportionately. ¢ The earlier in pregnancy discordancy develops, the more serious the sequelae.

¢ Ultrasonographic

monitoring of growth ¢ Fetal surveillance ¢ Delivery is usually not performed for size discordancy alone

PREVENTION OF PRETERM DELIVERY ¢  Bedrest ¢  Tocolytic

therapy for lung maturation ¢  Cerclage – NOT EFFECTIVE ¢  Corticosteroids

DELIVERY OF TWIN FETUSES

ANALGESIA AND ANESTHESIA

MANAGEMENT

Complications of labor and delivery ¢ preterm labor ¢ uterine contractile dysfunction ¢ abnormal presentation, prolapse of the umbilical cord ¢ premature separation of the placenta ¢ immediate postpartum hemorrhage

¢  EPIDURAL

¢  1.TWINS:

ANALGESIA-Pain relief,internal podalic version,cesarean section ¢  .

¢  CEPHALIC

CEPHALIC-VAGINAL/FORCEPS DELIVERY/ BREECH EXTRACTION OF SECOND TWIN ¢  NON CEPHALIC FIRST TWIN- CESAREAN DELIVERY ¢  CEPHALIC/BREECH-VAGINAL

¢  2.TRIPLETS

OR HIGHER ORDER GESTATION:: ¢  CESAREAN DELIVERY

CLASSIFICATION OF HYPERTENSIVE DISORDERS

HYPERTENSIVE DISORDERS IN PREGNANCY DEPARTMENT OF OBSTETRICS AND GYNECOLOGY FEU-NRMF INSTITUTE OF MEDICINE

PREECLAMPSIA Minimum criteria ¢  BP ≥ 140/90 mmHg after 20 weeks gestation ¢  Proteinuria ≥300mg/ 24 hours or ≥ 1+ dipstick

¢  ¢  ¢ 

¢  ¢  ¢  ¢  ¢ 

Increased certainty of preeclampsia BP ≥ 160/110mmHg after 20 weeks gestation Proteinuria 2.0 g/24 hours or ≥ 2 + dipstick Serum creatinine > 1.2mg/dl unless known to be previously elevated Platelets < 100,000/mm3 Microangiopathic hemolysis (increased LDH) Elevated ALT or AST Persistent headache or other cerebral or visual disturbances Persistent epigastric pain

COMPLICATING PREGNANCY (WORKING GROUP OF THE NHBPEP 2000):

¢ Gestational

Hypertension ¢ Preeclampsia ¢ Eclampsia ¢ Chronic Hypertension ¢ Superimposed Preeclampsia on Chronic hypertension

SEVERITY OF PREECLAMPSIA

GESTATIONAL HYPERTENSION ¢  BP ¢  No

≥ 140/90mmHg after 20 weeks AOG proteinuria

¢  BP

returns to normal before12 weeks’ postpartum transient hypertension

ECLAMPSIA

¢ Seizures

that cannot be attributed to other causes in a woman with preeclampsia

CHRONIC HYPERTENSION

SUPERIMPOSED PREECLAMPSIA (ON CHRONIC HYPERTENSION) ¢  New-onset

¢  BP

≥140/90 mm Hg before pregnancy or diagnosed before 20 weeks' gestation

or hypertension first diagnosed after 20 weeks' gestation and persistent after 12 weeks' postpartum

proteinuria 300 mg/24 hours in hypertensive women with no proteinuria

¢  A

sudden increase in proteinuria or blood pressure or platelet count < 100,000/mm3 in women with chronic hypertension

ETIOLOGY

POTENTIAL CAUSES:

¢  Exposed

¢  Abnormal

to chorionic villi for the first time.

¢  Superabundance

of chorionic villi( twins or hydatidiform mole)

¢  Preexisting ¢  Genetics

vascular disease.

RISK FACTORS ¢  nulliparous

women and ethnicity (African-American ethnicity) ¢  genetic predisposition ¢  environmental factors ¢  chronic hypertension ¢  multifetal gestation ¢  maternal age over 35 years ¢  obesity ¢  race

ABNORMAL TROPHOBLASTIC INVASION

trophoblastic invasion of uterine vessels ¢  Maternal maladaptation to cardiovascular or inflammatory changes of normal pregnancy ¢  Immunological intolerance between maternal and fetoplacental tissues ¢  Dietary deficiencies ¢  Genetic influences

PATHOPHYSIOLOGY

PATHOGENESIS:

¢ Vasospasm ¢ Endothelial

damage

HEMODYNAMIC CHANGES ¢  Blood

Volume - hemoconcentration

¢  Blood

and Coagulation - Thrombocytopenia results from platelet activation, aggregation, and consumption

HELLP SYNDROME H – Hemolysis ( LDH ) EL – Elevated liver enzymes ( AST or ALT) LP – Low Platelet count

“An indication for delivery”

LIVER ¢ 

BRAIN

periportal hemorrhage

¢ Subcapsular

rupture

hematoma and liver

PREDICTORS OF PREGNANCY INDUCED HYPERTENSION ¢  Roll

over test acid ¢  Fibronectin ¢  Coagulation activation ¢  Oxidative stress ¢  Cytokines ¢  Placental peptides ¢  Fetal DNA ¢  Uterine artery doppler velocimetry ¢  Uric

headaches and visual symptoms gross hemorrhage due to ruptured arteries principal postmortem lesions in eclampsia - edema - hyperemia - ischemia - thrombosis - hemorrhage Blindness (amaurosis)

KIDNEY ¢ 

¢  ¢  ¢ 

GFR (oliguria) uric acid creatinine Urine sodium

Anatomical Changes: glomerular capillary endothelial swelling PROTEINURIA

UTEROPLACENTAL PERFUSION ¢  Compromised

uteroplacental perfusion ¢  from vasospasm is the major ¢  culprit in increased perinatal morbidity and mortality

PREVENTION ¢  Dietary

manipulation supplementation ¢  Fish oil supplementation ¢  Antioxidant (Vit. C & E ) ¢  Low dose aspirin-PROVEN EFFECTIVE ¢  Calcium

“Delivery is the definitive cure for preeclampsia”

MANAGEMENT OBJECTIVES: 1. 

2.  3. 

Termination of pregnancy with the least possible trauma to mother and fetus. Birth of an infant who subsequently thrives. Complete restoration of health to the mother

¢  MANAGEMENT

IS BASED ON severity of the disease and gestational age of the fetus. disease is delivered at 34 weeks. disease at < 34 weeks EXPECTANT provided maternal and fetal conditions are good. ¢  Mild disease or non-severe disease: await spontaneous labor but with more frequent check-ups

ECLAMPSIA ¢  Preeclampsia

complicated by generalized tonic–clonic convulsions.

¢  Severe ¢  Severe

Differential Diagnosis: —  —  —  —  —  — 

TREATMENT 1. 

2. 

3. 

4. 

Control of convulsions using loading dose of magnesium sulfate(iv), followed either by a continuous infusion or intramuscular Intermittent intravenous or oral administration of an antihypertensive medication to lower blood pressure Avoidance of diuretics and limitation of intravenous fluid administration. Delivery.

ANALGESIA AND ANESTHESIA

TOXICOLOGY OF MAGNESIUM SULFATE ¢  Magnesium

intoxication :WHAT TO MONITOR? - urine output is adequate - the patellar or biceps reflex is present -no respiratory depression ¢  Therapeutic level: 4 to 7 mEq/L ¢  Toxic Levels: - 10 mEq/L Patellar reflexes disappear - >10 mEq/L respiratory depression develops, - >12 mEq/L respiratory paralysis and arrest follow ¢  AntIdote: Calcium gluconate, 1 g intravenously

ANTIHYPERTENSIVE MEDICATIONS ¢  Hydralazine-5mg

every 15-20 mins

¢  Labetalol

- nonselective ß-blocker ¢  Nifedipine

- 10-mg oral dose Methyldopa-drug of choice for oral medications Diuretics-for patients with pulmonary edema

DESIGN OF FORCEPS §  § 

Two crossing branches four components: ú 

¢ Epidural

Epilepsy Encephalitis Meningitis Cerebral tumor Cysticercosis Ruptured cerebral aneurysm

analgesia

ú  ú  ú  § 

Blade – fenestrated or solid Shank – connects the handle and the blade Lock – holds the forceps together Handle – to grip the forceps

two curves ú  ú 

cephalic curve conforms to the shape of the fetal head pelvic curve corresponds more or less to the axis of the birth canal   

Some varieties are fenestrated or pseudofenestrated to permit a firmer hold on the fetal head

 TYPES OF FORCEPS SIMPSON FORCEPS §  Most common forceps §  Parallel shanks §  English-style lock § 

molded head (nulliparous)

KIELLAND FORCEPS ¢  sliding lock, ¢  minimal pelvic curvature, and light weight ¢  for deep transverse arrest

TUCKER-MCLANE FORCEPS •  blade is solid and the shank is narrow. •  English lock •  rounded head (multiparous)

FUNCTIONS OF OBSTETRIC FORCEPS PIPER FORCEPS —  —  — 

blade is similar to simpson shank is longer it has a double pelvic curve for aftercoming head in breech presentation

BARTON FORCEPS ¢  good forceps for rotation of head in transverse arrest

1. 

Traction The direction of the traction must be along the pelvic curvature

ú 

2. 

ú 

Rotation Carried out best in the mid pelvis

     

CLASSIFICATION OF FORCEPS DELIVERIES Procedure Criteria Scalp is visible at introitus without separating the labia Outlet Fetal skull has reached pelvic floor Sagittal suture is in anteroposterior diameter or right or left occiput anterior or posterior position Fetal head is at or on the perineum Rotation does not exceed 45 degrees Low

Midpelvic High

1.  Leading point of fetal skull is at station ≥+2 cm, and not on pelvic floor 2.  Rotation is 45 degrees or less (left or right occiput anterior to occiput anterior, or left or right occiput posterior to occiput posterior) 3.  Rotation is greater than 45 degrees Station above +2 cm but head is engaged Not included in classification

INDICATIONS FOR FORCEPS Fetal indications 1.  prolapsed of the umbilical cord 2.  premature separation of the placenta 3.  non-reassuring fetal heart rate pattern

CONTRAINDICATIONS TO USE FORCEPS

Maternal indications 1.  2.  3.  4.  5.  6.  7. 

heart disease hypertensive condition pulmonary injury or compromise intrapartum infection neurologic condition exhaustion prolonged second stage

1.  2.  3.  4.  5. 

absence of proper indication incompletely dilated cervix marked cephalo-pelvic disproportion unengaged fetal head lack of experience on the part of the operator

PREREQUISITES FOR FORCEPS APPLICATION 1.  2.  3.  4.  5.  6. 

PREPARATION FOR FORCEPS DELIVERY

Engaged head vertex or chin anterior position must be precisely known Fully dilated cervix Ruptured membranes no CPD

§ 

ANESTHESIA: pudendal block / regional analgesia or general anesthesia bladder should be emptied

¢ 

the head of the fetus is perfectly grasped only when the long axis of the blades corresponds to the OCCIPITOMENTAL diameter

OUTLET FORCEPS DELIVERY The forceps are applied as follows: §  Two or more fingers of the right hand are introduced inside the left posterior portion of the vulva and into the vagina beside the fetal head. §  The handle of the left branch is then grasped between the thumb and two fingers of the left hand and the tip of the blade is gently passed into the vagina between the fetal head and the palmar surface of the fingers of the right hand

§ 

For application of the right blade, two or more fingers of the left hand are introduced into the right, posterior portion of the vagina to serve as a guide for the right blade, which is held in the right hand and introduced into the vagina as described for the left blade. After positioning, the branches are articulated.

MORBIDITY FROM FORCEPS OPERATIONS ¢  Forceps

have been disarticulated and removed, and ¢  MODIFIED RITGENS maneuver (arrow) is used to complete delivery of the head.

Maternal Morbidity 1.  lacerations of the vulva, vagina and cervix and extension of episiotomy 2.  urinary and rectal incontinence 3.  infection

Fetal Morbidity 1.  poor apgar score 2.  cephalhematoma 3.  caput 4.  facial mark or injury 5.  Erb palsy 6.  Fractured clavicle 7.  Elevated bilirubin 8.  Retinal hemorrhage

¢  Horizontal

traction is exerted until the perineum begins to bulge AND episiotomy may be performed if indicated

TRIAL FORCEPS ¢  the

operator attempts delivery with the full knowledge that the vaginal delivery may not be successful

FAILED FORCEPS ¢  Failure

of application: the forceps cannot be applied properly to the fetal head

¢  Failure

of Extraction: the forceps are applied but despite an all-out effort, delivery cannot be accomplished

Causes ¢  Disproportion ¢  Malposition ¢  Cervix not fully dilated ¢  Constriction ring ¢  Premature interference

VACUUM EXTRACTION Vacuum Extraxtor (Ventouse) § 

§  §  § 

INDICATIONS AND PREREQUISITES Generally, the indications and prerequisites for the use of the vacuum extractor for delivery are the same as for forceps delivery Fetal indications 1.  prolapsed of the umbilical cord 2.  premature separation of the placenta 3.  non-reassuring fetal heart rate pattern

CONTRAINDICATIONS 1. 

Maternal indications 1.  2.  3.  4.  5.  6.  7. 

2. 

heart disease hypertensive condition pulmonary injury or compromise intrapartum infection neurologic condition exhaustion prolonged second stage

3.  4.  5.  6.  7.  8. 

operator inexperience inability to assess fetal position high station suspicion of cephalopelvic disproportion face or other nonvertex presentations fetal coagulopathy macrosomia recent scalp blood sampling

TECHNIQUE §  § 

proper cup placement 3 cm in front of the posterior fontanelle toward the face

§ 

Entrapment of maternal soft tissues- lacerations and hemorrhage and cup "pop-off."

§ 

full circumference of the cup should be palpated both before and after the vacuum and prior to traction. traction should be intermittent and coordinated with maternal expulsive efforts

Abandon procedure if there are 3 pop offs

Mityvac

PREREQUISITES FOR VACUUM EXTRACTION 1.  2. 

3. 

4.  5.  6. 

head must be engaged fetus must present as a vertex or by the face with the chin anterior position of the fetal head must be precisely known cervix must be completely dilated membranes must be ruptured no suspected cephalic–pelvic disproportion

COMPLICATIONS

§  § 

metal cup or soft cup vacuum extractors high-pressure vacuum Silastic cup Mityvac instrument

Silastic cup vacuum

§  §  §  §  §  §  §  §  §  §  §  § 

scalp lacerations and bruising subgaleal hematomas cephalohematomas intracranial hemorrhage neonatal jaundice subconjunctival hemorrhage clavicular fracture shoulder dystocia injury of sixth and seventh cranial nerves Erb palsy retinal hemorrhage fetal death

CESAREAN DELIVERY AND PERIPARTUM HYSTERECTOMY:

COMPARISON OF VACUUM EXTRACTION WITH FORCEPS Vacuum Extraction ¢  Increase incidence of neonatal jaundice ¢  Shoulder dystocia and cephalhematoma is doubled Forceps Delivery ¢  Higher frequency of maternal trauma and blood loss ¢  More 3rd and 4th degree laceration

CESAREAN DELIVERY AND PERIPARTUM HYSTERECTOMY

INDICATIONS FOR CESAREAN DELIVERY

CANDIDATES FOR VBAC

Cesarean Delivery

birth of a fetus through incisions in the abdominal wall (laparotomy) and the uterine wall (hysterotomy).

Cesarean Hysterectomy

Hysterectomy that is performed at the time of cesarean delivery

Postpartum Hysterectomy

Hysterectomy that is done within a short time after vaginal delivery

INDICATIONS FOR PRIMARY CS ¢  DYSTOCIA-

Prior cesarean delivery

No more than 1 prior low-transverse CS delivery

Labor dystocia

Clinically adequate pelvis

Fetal distress

No other uterine scars or previous uterine rupture

Breech presentation

most frequent indication DISTRESS ¢  BREECH PRESENTATION ¢  FETAL

Physician immediately available throughout active labor who is capable of monitoring labor and performing emergency CS Availability of anesthesia and personnel for emergency CS

TECHNIQUE FOR CESAREAN DELIVERY ABDOMINAL INCISIONS

TECHNIQUE FOR CESAREAN DELIVERY

TECHNIQUE FOR CESAREAN DELIVERY ABDOMINAL INCISION

midline vertical or a suprapubic transverse incision

ABDOMINAL INCISION VERTICA L INCISION

Infraumbilical midline vertical incision

TRANSVERSE INCISION

Modified Pfannenstiel incision

ADVANTAGE Cosmetic

paramedian or midtransverse incision-RARELY

Stronger incision Less dehiscence or hernia Advantage Can be rapidly extended Quickest to make

DISADVANTAGE Suboptimal exposure S Reentry is more difficult due to scarring MAYLARD INCISION

rectus muscle divided w/ scissors and scalpel useful in women with scarring resulting from previous Pfannensteil incision

TECHNIQUE FOR CESAREAN DELIVERY

TECHNIQUE FOR CESAREAN DELIVERY

TECHNIQUE FOR CESAREAN DELIVERY

UTERINE INCISION

UTERINE INCISION

UTERINE INCISION

KERR

lower uterine segment transverse incision

KRONIG

Operation of choice Advantages easier to repair less incidence of rupture

cesarean delivery that is performed on a woman who has just died

Transverse lie of large fetus Placenta previa with anterior implantation

May extend to the cervix and bladder

Very small fetuses with lower uterine segment not thinned out massive maternal obesity

More likely to rupture

PERIPARTUM HYSTERECTOMY INDICATION uterine atony (most common ) S Lower uterine segment bleeding

Laceration of major uterine vessels Placenta accreta Large myoma Severe cervical dysplasia or CIS Intrauterine infection

Vertical incision in the body of the uterus

Indications LOWER SEGMENT CANT BE EXPOSED 1. bladder adhesions 2. myoma 3. invasive CA of the cervix

Disadvantage extensive dissection of the s bladder

LESS adherence to bowel or omentum

POSTMORTEM CESAREAN DELIVERY

Low segment vertical incision

CLASSICAL INCISION

PERIPARTUM HYSTERECTOMY

MAJOR Increased blood COMPLICATIO loss NS Possibility of urinary tract damage Increased morbidity associated with emergency hysterectomy

Grossly defective scar

QUESTIONS: ¢  1.A

G1P1 PU 13 weeks has an ultrasound result twin pregnancy with single chorion and 2 amnion.When does the division of the monozygotic twin occurred ? ¢  a.0-4 days ¢  b.4-8 days ¢  c.8-12 days ¢  d.>13 days

¢  2.Which

of the following must NOT be done in a monoamnionic monochorionic twins? ¢  a.Daily CTG at starting at viability ¢  b.steroids at 26-28 weeks ¢  c.Deliver at 38 weeks ¢  d.terminate by CS

¢  3.Which

of the following characterizes the recipient in twin to twin transfusion// ¢  a.anemic ¢  b.hyperbiliribunemia ¢  c.IUGR ¢  d.oligohydramnios

¢  4.Which

of the following presentations in multifetal pregnancy can be delivered vaginally in multiparous patient? ¢  a.twin breech-cephalic ¢  b.twin-cephalic breech ¢  c.twin-cephalic-transverse ¢  d.triplets all cephalic

¢  5.21y/o

G1P0 PU 32 weeks cephalic,complaining of headache.BP-160/100 mmhg.Urine protein ++ +.What is the diagnosis? ¢  a.gestational hypertension ¢  b.chronic hypertension ¢  c.transient hypertension ¢  d.severe pre eclampsia

¢  6.36

¢  7.Which

of the following is the most effective in the prevention of pre eclampsia? ¢  a.low dose aspirin ¢  b.high dose calcium ¢  c.fish oil ¢  d.antioxidants

¢  8.G2P

0 PU 35 weeks complained of epigastric pain .BP-190/100 mmhg.Lab test revealed low platelets and increased LDH.What is the definitive management of this patient? ¢  a.control hypertension with hydralazine ¢  b.prevent convulsion with MG SO4 ¢  c.weekly surveillance testing ¢  d.terminate pregnancy

¢  9.Which

¢  10.In

¢  11.How

¢  12.Which

what diameter of the pelvis will the forcep fits during application? ¢  a.biparietal ¢  b.occipitofrontal ¢  c.occipitomental ¢  d.suboccipitobregmatic

many pop offs during vacuum extraction before you will abandon the procedure? ¢  a.1 ¢  b.2 ¢  c.3 ¢  d.4

y/o G1P0 PU 36 weeks was admitted because of blurring of vision.BP-150/100 mmhg,urine protein +++.Lab tests revealed low platelets,increased LDH,SGPT and alkaline phosphatase.What is the complete diagnosis? ¢  a.Pre eclampsia non severe ¢  b.Pre eclampsia,severe ¢  c.Pre eclampsia,severe, HELLP syndrome ¢  d.Pre eclampsia ,severe,DIC

forcep is described to have a longer shank and a double pelvic curve? ¢  a.bartons ¢  b.pipers ¢  c.simpsons ¢  d.kiellands

of the following will qualify a patient for a vaginal birth after a cesarean section? ¢  a.one previous Classical CS ¢  b.no previous uterine rupture in last 2yrs ¢  c.can be performed in a lying in with physician available ¢  d.The obstetrician and anesthesiologist must be available

¢  13.What

is the MOST frequent indication for primary CS? ¢  a.malpresentation ¢  b.dystocia ¢  c.fetal distress ¢  d.maternal illness

¢  14.Which

of the following is a disadvantage of pfannesteil incision? ¢  a.weak ¢  b.more dehiscence ¢  c.difficult re entry ¢  d.faulty healing

¢  15.What

PUERPERAL INFECTION

PUERPERAL FEVER

¢  any

¢  temperature

bacterial infection of the genital tract after delivery

is the most frequent indication for CS hysterectomy? ¢  a.atony ¢  b.laceration of uterine vessels ¢  c.accreta ¢  d.myoma

– 38.0° C (100.4° F) or higher at any 2 of the first 10 days postpartum, exclusive of the first 24 hours and to be taken by mouth by a standard technique at least 4 times daily

PUERPERAL INFECTION

PUERPERAL FEVER Incidence : ¢  Puerperal Sepsis : 32.1% ¢  Intrapartum Sepsis (Chorioamnionitis) : 2.1% ¢  Postabortal Sepsis : 1.5%

DIFFERENTIAL DIAGNOSIS OF PUERPERAL FEVER ¢  Genital

Tract Infection Causes

¢  Extragenital —  —  —  —  — 

Respiratory Complications Pyelonephritis Breast Engorgement Superficial or Deep Venous Thrombophlebitis Incisional Wound Abscess

GENITAL TRACT INFECTION Risk Factors : ¢  prolonged rupture of membranes ¢  intrauterine electronic monitoring ¢  extensive vaginal / uterine manipulation

RESPIRATORY COMPLICATIONS • 

most often seen within the first 24 hours ,most often seen in women delivered by cesarean section –  – 

atelectasis, aspiration pneumonia or occasionally bacterial pneumonia Atelectasis is prevented by routine coughing, early ambulation, deep breathing every 4 hours for 24 hours

SUPERFICIAL OR DEEP VENOUS THROMBOPHLEBITIS •  •  • 

minor temperature elevations, painful, swollen leg calf tenderness or occasionally femoral triangle area tenderness

¢ treated

heparin

with intravenous

PYELONEPHRITIS

BREAST ENGORGEMENT

• 

bacteriuria, pyuria, costovertebral angle tenderness and spiking temperature

• 

brief temperature elevation rarely exceeds 39.0 C and no longer than 24 hours

• 

nausea and vomiting developing later

• 

Bacterial Mastitis- fever develops later,persistent and with signs and symptoms of breast infection

• 

Pyuria and bacteriuria

POSTPARTUM UTERINE INFECTION

PREDISPOSING FACTORS

¢ endometritis,

• 

endomyometritis, endoparametritis ¢ the route of delivery is the single most significant risk factor for the development of postpartum uterine infection

•  •  •  •  • 

Route of delivery Labor Rupture of membranes Number of vaginal examinations Low socioeconomic status Others –  –  – 

Anemia Nutrition Sexual intercourse

ROUTE OF DELIVERY

LABOR

RUPTURE OF MEMBRANES

¢  Vaginal

¢  prolonged

¢  if

Delivery – 2-3% Risk factors —  —  —  — 

prolonged rupture of membranes and labor multiple cervical examination internal fetal monitoring Intra – amniotic infection

¢  Cesarean

Delivery – 5-6%

duration of labor = increased cervical and vaginal examinations ¢  postpartum morbidity after cesarean section

> 6 hours = (+) pathogenic bacteria developed endometritis

¢  95%

NUMBER OF VAGINAL EXAMINATION

LOW SOCIOECONOMIC STATUS

OTHERS

¢  increased

¢  possible

• 

postpartum endometritis

causes : differences in vaginal flora, hygiene and nutrition

Anemia – 

• 

Nutrition – 

• 

•  •  •  •  •  •  •  •  •  •  •  •  •  •  •  •  •  •  • 

Aerobes    Group A, B, and D streptococci   Enterococcus   Gram-negative bacteria—Escherichia coli, Klebsiella, and Proteus species    Staphylococcus aureus    Staphylococcus epidermidis    Gardnerella vaginalis  Anaerobes    Peptococcus species    Peptostreptococcus species    Bacteroides fragilis group    Prevotella species    Clostridium species    Fusobacterium species    Mobiluncus species  Other    Mycoplasma species    Chlamydia trachomatis    Neisseria gonorrhoeae

ANTIMICROBIAL REGIMENS FOR PELVIC INFECTION FOLLOWING CESAREAN DELIVERY Regimen

Comments

Clindamycin 900 mg +gentamicin 1.5 "Gold standard," 90–97% efficacy, mg/kg, q8h intravenously once-daily gentamicin dosing acceptable plus ampicillin

Added to regimen with sepsis syndrome or suspected enterococcal infection

Clindamycin+aztreonam

Gentamicin substitute with renal insufficiency

Extended-spectrum penicillins

Piperacillin, ampicillin/sulbactam

Extended-spectrum cephalosporins

Cefotetan, cefoxitin, cefotaxime

Imipenem + cilastatin

Reserved for special indications

cell – mediated immunity is impaired in malnourished

Sexual Intercourse – 

BACTERIA COMMONLY RESPONSIBLE FOR FEMALE GENITAL INFECTIONS

does not predispose to infection but normal hemoglobin may prevent infection by increasing transferrin(with significant antibacterial function)

Not clearly demonstrated

METRITIS

¢ polymicrobial ¢  Pathogenesis —  —  — 

:

involves the placental implantation site and the deciduas and adjacent myometrium Foul, profuse, bloody and frothy discharge Leukocytic infiltration

WOUND INFECTIONS

NECROTIZING FASCIITIS

Risk factors: ¢  obesity ¢  diabetes ¢  corticosteroid therapy ¢  immunosuppression ¢  anemia ¢  poor hemostasis with hematoma formation

¢  rare

but fatal abdominal incisions following cesarean delivery, or may complicate episiotomy or perineal laceration ¢  significant tissue necrosis ¢  involve

NECROTIZING FASCIITIS

NECROTIZING FASCIITIS

Risk factors for fasciitis ¢  diabetes, obesity, and hypertension ¢  may be caused by a single virulent bacterial species such as group A beta-hemolytic streptococcus, but more commonly they are polymicrobial

Treatment ¢  Clindamycin given with a beta-lactam antimicrobial - most effective regimen ¢  promptly debriding wide margins of the fascial incision

TOXIC SHOCK SYNDROME

TOXIC SHOCK SYNDROME ¢  Staphylococcus

• 

• 

• 

acute febrile illness with severe multisystem derangement fever, headache, mental confusion, diffuse macular erythematous rash, subcutaneous edema, nausea, vomiting, watery diarrhea, marked hemoconcentration renal failure à hepatic failure, DIC à circulatory collapse

toxin – 1

NECROTIZING FASCIITIS ¢  treatment

: wide debridement of all infected tissue, split thickness skin grafts ¢  mortality -50% even if aggressive excision is performed

aureus – toxic shock syndrome

¢  Therapy

: SUPPORTIVE, anti-staphylococcal antimicrobials, massive fluid replacement, mechanical ventilation with PEEP, renal dialysis

CARDIOVASCULAR DISEASE

HEART DISEASE IN PREGNANCY There are certain peaks of cardiac activity during pregnancy: Early third trimester

¢  ECHOCARDIOGRAPHY ¢  CHEST

XRAY

¢  ELECTROCARDIOGRAM

Durin g labor

During puerperium

DIAGNOSTIC STUDIES

During delivery It is during these periods when cardiac failure is likely to occur.

CLINICAL CLASSIFICATION

Class I

• Uncompromised (no limitation of physical activity): • These women do not have symtoms of cardiac insuffuciency or experience anginal pain

CLINICAL CLASSIFICATION

• Slight limitation of physical activity:

Class II

• These women are comfortable at rest, but if ordinary physical activity is undertaken, discomfort results in the from of excessive fatigue, palpitation, dyspnea, or anginal pain

Class III

• Marked limitation of physical activity: • The women are comfortable at rest, but less than ordinary activity causes excessive fatigue, palpitation, dysnea, or anginal pain

Class IV

• Severely Compromised (inability to perform any physical activity without discomfort): • Symptoms of cardiac insufficiency or angina even at rest, and if any activity is undertaken, discomfort is increased

FACTORS THAT WILL CAUSE HEART FAILURE IN THE PUERPERAL PERIOD

CONTRACEPTION

¢  Anemia

¢  Progestin

only pills thromboembolism ¢  IUD-contraindicated /causes subacute bacterial endocarditis ¢  Tubal Sterilization-delayed until patient can ambulate

MANAGEMENT OF CLASS I & II Vaginal delivery

Mode of Delivery: Relief from pain:

•  epidural anesthesia •  GA •  avoid subarachnoid block

MANAGEMENT OF CLASS III AND IV DISEASE

¢  OCP-causes ¢  Hemorrhage ¢  Infection

¢ 

thromboembolism

ANTIMICROBIAL PROPHYLAXIS

ANTICOAGULATION OF PREGNANT WOMEN WITH CARDIAC DISORDERS

TO BE GIVEN 30-60 MINUTES PRIOR TO DELIVERY… Ampicillin 2gms IV or Amoxicillin 2 gms oral

•  usually for patients with mechanical prosthetic valves

If penicillin sensitive: Cefazolin or Ceftriaxone 1 gm IV If with history of anaphylaxis: Clindamycin 600 mg IV If with enterococcal infection: add vancomycin

q severe cardiac disease should consider pregnancy interruption. q  prolonged hospitalization or bed rest q Epidural analgesia q Vaginal delivery (Forceps/vacuum) q Cesarean delivery is limited to obstetrical indications.

Unfractionated Heparin • given at 6-12 weeks. Then • resumed at 36 weeks and discontinued before delivery

Warfarin • started at 13 weeks and discontinued at 36 weeks • resumed postpartum

PULMONARY DISORDERS

ASTHMA

ASTHMA

¢ seen

frequently during pregnancy ¢ chronic inflammatory airway disorder with a major hereditary component.

The hallmarks of asthma are: q reversible airway obstruction tenacious mucus, & mucosal edema q airway inflammation

PATHOPHYSIOLOGY

Because F-series prostaglandins and ergonovine exacerbate asthma, should be avoided if possible

EFFECTS OF PREGNANCY ON ASTHMA

EFFECTS OF ASTHMA ON PREGNANCY

TREATMENT

¢ There

¢ Unless

Treatment depends on the severity of disease

is no evidence that pregnancy has a predictable effect on underlying asthma.

MANAGEMENT OF ACUTE ASTHMA

¢ 1st

line therapy: Beta Adrenergic agonists

¢ MOA:

Bronchial smooth muscle relaxation

Terbutaline

Epinephrine

Albuterol

Isoproterenol

Isoetharine

Metaproterenol

there is severe disease, pregnancy outcomes are generally excellent

REGIMENS RECOMMENDED FOR

agonists help to abate bronchospasm, corticosteroids treat the inflammatory component

LABOR AND DELIVERY

OUTPATIENT MANAGEMENT

q Maintenance medications are continued through delivery

q For mild asthma, inhaled agonists q  For persistent asthma, inhaled corticosteroids

100 mg of hydrocortisone given intravenously every 8 hours during labor and for 24 hours after delivery

LABOR AND DELIVERY Epidural is ideal

Avoid tracheal intubation Oxytocin and PGE2 – ok

PNEUMONIA

INCIDENCE AND CAUSES ¢ Pregnancy

Any pregnant woman suspected of having pneumonia should undergo chest radiography

itself does not predispose to pneumonia

¢  S.

pneumoniae is the most common cause

PGF2 and ergotamine derivatives – NOT OK CHEST RADIOGRAPHS IN A PREGNANT WOMAN WITH RIGHT LOWER LOBE PNEUMONIA. A. COMPLETE OPACIFICATION OF THE RIGHT LOWER LOBE (ARROWS) IS CONSISTENT WITH THE CLINICAL SUSPICION OF PNEUMONIA. B. OPACIFICATION (ARROWS) IS ALSO SEEN ON THE LATERAL PROJECTION.

DIAGNOSIS

MANAGEMENT ¢ Antimicrobial

empirical

q Symptoms : cough, dyspnea, sputum production, and pleuritic chest pain. q Chest radiography is essential for diagnosis

PREGNANCY OUTCOME WITH PNEUMONIA ¢ Maternal ¢ Preterm ¢ PPROM

Mortality: 0.8%

labor

treatment is

macrolide—azithromycin, clarithromycin, or erythromycin.

¢ 

PREVENTION

q Pneumococcal vaccine is 60- to 70percent protective q Vaccine is not recommended for healthy pregnant women q  Recommended for immunocompromised patients

INFLUENZA PNEUMONIA

q RNA viruses: Influenza A and B infection is self-limited. Pneumonia is the most common complication, and it is difficult to distinguish from bacterial pneumonia.

CHEST RADIOGRAPH OF A 30-WEEK PREGNANT WOMAN WITH CONFIRMED INFLUENZA

A PNEUMONIA.

PREVENTION

¢ Vaccination

for influenza A is recommended during pregnancy

TUBERCULOSIS AND PREGNANCY Effect on Pregnancy ¢  preterm delivery ¢  low birthweight ¢  growth-restricted infants ¢  ↑ perinatal mortality

Without antituberculosis therapy, pregnancy likely has adverse effects on the course of active tuberculosis

MANAGEMENT ¢  Supportive

treatment with antipyretics and bed rest is recommended for uncomplicated influenza.

MANAGEMENT ¢ Oseltamivir

is given orally, 75 mg twice daily, or zanamivir is given by inhalation, 10 mg twice daily

TUBERCULOSIS

DIAGNOSIS OF PTB

¢ MOT:

¢ Chest

inhalation of Mycobacterium tuberculosis; a granulomatous pulmonary reaction

¢ Clinical

manifestations:

cough with minimal sputum production —  low-grade fever —  hemoptysis —  weight loss

X-ray:

—  infiltrative

pattern

—  cavitation —  mediastinal

lymphadenopathy

— 

TUBERCULOSIS SCREENING

¢ Purified

protein derivative (PPD) — If

negative (5mm) à CXR

AFB Culture AFB Smear

TUBERCULOSIS

¢ Latent ¢ Active

ACTIVE INFECTION

Drug regimen: Isoniazid, Rifampin, and Ethambutol Isoniazid resistant tuberculosis - Pyrazinamide is added to initial regimen, given for 9 months Breastfeeding is not prohibited during antituberculosis therapy Pyridoxine, 25mg/day orally, to decrease hepatic toxicity due to isoniazid

ACTIVE INFECTION

NEONATAL TUBERCULOSIS

RENAL DISEASES DURING PREGNANCY

¢ aminoglycosides—

streptomycin, kanamycin, amikacin, and capreomycin— are ototoxic to the fetus and are contraindicated

NEONATAL TUBERCULOSIS ¢ Congenital

Tuberculosis

—  bacilli

blood placenta of infected secretion during delivery

—  aspiration

ASSESMENT OF RENAL DISEASE IN PREGNANCY

¢ Urinalysis ¢ Ultrasound ¢ Intravenous

Neonatal infection is unlikely if the mother with active disease has been treated before delivery

URINARY TRACT INFECTIONS v  Most

common bacterial infections in pregnancy v  90 percent are caused by Escherichia coli  

Pyelography

¢ Cystoscopy ¢ MRI

of renal masses ¢ Renal biopsy ( usually postponed until pregnancy is completed)  

ASYMPTOMATIC BACTERIURIA v 

persistent, actively multiplying bacteria in women who have no symptoms

also more common in diabetics DIAGNOSIS: A clean-voided specimen containing more than 100,000 organisms per mL

v 

Table 48–1. Antimicrobial Agents Used for Treatment of Pregnant Women with Asymptomatic Bacteriuria

Single-dose treatment Amoxicillin, 3 g Ampicillin, 2 g Cephalosporin, 2 g Nitrofurantoin, 200 mg Trimethoprim-sulfamethoxazole, 320/1600 mg 3-day course Amoxicillin, 500 mg three times daily Ampicillin, 250 mg four times daily Cephalosporin, 250 mg four times daily Ciprofloxacin, 250 mg twice daily Levofloxacin, 250 mg daily Nitrofurantoin, 50 to 100 mg four times daily; 100 mg twice daily Trimethoprim-sulfamethoxazole, 160/800 mg two times daily

CYSTITIS & URETHRITIS

ACUTE PYELONEPHRITIS v 

v  Chlamydia

trachomatis, Frequency, urgency, dysuria, and sterile pyuria , urine culture with no growth. v  Management: AZITHROMYCIN

v 

v 

ACUTE PYELONEPHRITIS CLINICAL FINDINGS:

is the most common serious medical complication of pregnancy . urosepsis was the leading cause of septic shock

ACUTE PYELONEPHRITIS v  Differential

diagnosis: labor, chorioamnionitis, appendicitis, placental abruption or infarcted myoma v  Monitor plasma creatinine

more common in the second trimester , nullipara and young age

Table 48–2. Management of the Pregnant Woman with Acute Pyelonephritis

ANTIMICROBIAL THERAPY FOR ACUTE PYELONEPHRITIS v  Ampicillin

1. Hospitalization

plus gentamicin

2. Urine and blood cultures

anorexia, nausea, and vomiting. Fever as high as 40 C v  Tenderness at the costovertebral angles. v  leucocytosis some in clumps. v  v 

3. Hemogram, serum creatinine, and electrolytes 4. Monitor vital signs frequently, including urinary output; consider indwelling catheter 5. Intravenous crystalloid to establish urinary output to >/=50 mL/hr-cornerstone*** 6. Intravenous antimicrobial therapy 7. Chest radiograph if there is dyspnea or tachypnea

   

8. Repeat hematology and chemistry studies in 48 hours 9. Change to oral antimicrobials when afebrile

v  Cefazolin

or ceftriaxone

v  Extended

spectrum antibiotic & gentamicin – excellent vs E. Coli v  Oral therapy after discharge for 7 to 14 days v  Cephalosporins

10. Discharge when afebrile 24 hours; consider antimicrobial therapy for 7 to 10 days 11. Urine culture 1 to 2 weeks after antimicrobial therapy completed

THYROID FUNCTION CHANGES DURING PREGNANCY: ENDOCRINE DISORDERS IN PREGNANCY

DEPARTMENT OF OBSTETRICS AND GYNECOLOGY Far Eastern University – Nicanor Reyes Medical Foundation

¢ 

thyroid binding globulin

¢ 

daily thyroxine (T4) and T3

¢ Thyroid

Function Tests to request during pregnancy - TSH, free T4, free T3

DIAGNOSTIC SIGNS OF HYPERTHYROIDISM/ THYROTOXICOSIS: ¢  Severe

tachycardia sleeping pulse rate thyromegaly exophthalmos failure to gain weight serum free T4 and TSH

¢  elevated ¢  ¢  ¢  ¢ 

HYPERTHYROIDISM / THYROTOXICOSIS AND PREGNANCY

TREATMENT: MEDICAL ¢  Thioamide drugs: ¢  1. propylthiouracil-preferred because it partially inhibits the conversion of T4 to T3 and it crosses the placenta less readily.

Grave’s Disease - an autoimmune process due to thyroid stimulating antibodies.

¢  Ablation

with radioactive iodine is CONTRAINDICATED during pregnancy

¢  Maternal

thyroidectomy – NOT DONE DUE TO BLEEDING

¢  2.methimazole.

¢ 

S/E of methimazole: esophageal atresia choanal atresia aplasia cutis

HYPERTHYROIDISM / THYROTOXICOSIS AND PREGNANCY

HYPERTHYROIDISM / THYROTOXICOSIS AND PREGNANCY Thyroid Storm and Heart Failure Management: - propylthiouracil - iodide (inhibits thyroidal release of T3/T4) IV, oral (SSKI), Lugol solution - Lithium carbonate ( if with allergy to iodide) - Dexamethasone ( to further inhibit peripheral conversion of T4 to T3) - B-blocker (propranolol) to control tachycardia

Thyroid Storm and Heart Failure Thyroid Storm – an acute, lifethreatening, hypermetabolic state Heart Failure – due to cardiomyopathy

HYPOTHYROIDISM ¢  Due

to glandular destruction by autoantibodies

HYPOTHYROIDISM ¢ 

¢  ¢  Overt

hypothyroidism: TSH, FT3 and FT4

associated with infertility high incidence of preeclampsia, placental abruption low birth weight & stillborn infants.

FETAL AND NEONATAL EFFECTS ¢ 

goitrous thyrotoxicosis caused by placental transfer of thyroid stimulating antibodies or goitrous hypothyroidism due to thioamides treatment(less common)

¢  Hydrops

and fetal demise

HYPOTHYROIDISM ¢  EFFECTS ¢  -

¢ 

IN FETUS & INFANTS:

neuropsychological development.

impaired psychomotor development.

PREGNANCY INDUCED STATE OF PERIPHERAL RESISTANCE TO INSULINDUE TO:

HYPOTHYROIDISM ¢  MANAGEMENT:

GESTATIONAL DIABETES MELLITUS

Thyroxine replacement

Iodine Deficiency: RDA: 220 ug / day Mild deficiency: intelectual impairment Severe deficiency: endemic cretinism

¢  Increased

insulin response to glucose ( increase plasma level & duration)

¢  Reduced

peripheral uptake of glucose ( increased plasma level duration)

Congenital Hypothyroidism: Detection is part of newborn screening

¢  Suppressed

SCREENING: ¢  screening

for gestational diabetes - performed between 24 and 28 weeks

FETAL EFFECTS: MATERNAL EFFECTS:

50-g oral glucose challenge test is followed by a diagnostic 100-g oral glucose tolerance test if result IS MORE THAN 140 mg /dl

¢  Miscarriage ¢  Preterm

¢  Preeclampsia ¢  a

glucagon response.

delivery Fetal growth ¢  Fetal malformation ¢  Hydramnios ¢  Altered

bacterial infection fetus ¢  Hydramnios ¢  Increase maternal mortality ¢ 

¢  Macrosomic

NEONATAL MORTALITY AND MORBIDITY ¢  Respiratory — 

Distress Syndrome

Delayed fetal lung maturation

¢  Hypoglycemia — 

hyperplasia of the fetal -islet cells induced by chronic maternal hyperglycemia.

¢  Hypocalcemia

¢  Hyperbilirubinemia

and Polycythemia

¢  Cardiomyopathy — 

Hypertrophic cardiomyopathy

¢  Long-Term ¢  Inheritance

Cognitive Development of Diabetes

MANAGEMENT: ¢  Insulin therapy - when standard dietary management does not maintain the: ¢  FBS- less than 95 mg/dL or the 2-hour postprandial less than 120 mg/dL. ¢  exercise

POSTPARTUM SURVEILLANCE OF GDM PATIENTS: ¢  75-g

oral glucose tolerance test at 6 to 12 weeks after delivery.

MATERNAL EFFECTS OF OVERT DIABETES: ¢  Diabetes

nephropathy retinopathy ¢  Diabetic neuropathy ¢  Preeclampsia ¢  Ketoacidosis ¢  Infection ¢  Diabetic

¢  Low

dose OCP –SAFE FOR GDM PATIENTS

 

MANAGEMENT OF OVERT DIABETES: American Diabetes Association has defined optimal preconceptional glucose control using insulin: FBS-70 to 100 mg/dL :postprandial values of less than 140 mg/dL :less than 120 mg/ dL at 1 and 2 hours

FETAL EFFECTS OF OVERT DIABETES ¢  Perinatal

losses Macrosomia ¢  Abortion ¢  Preterm delivery at 34 weeks ¢  Malformations ¢  Unexpected fetal demise ¢  Hydramnios ¢  Fetal

MANAGEMENT OF OVERT DIABETES:

¢  The

¢  Hospitalization

- Maternal Alpha feto protein – 16-20 weeks - Congenital anomaly screening at 18-20 weeks. -Ultrasound later in pregnancy for macrosomia or IUGR

¢  Folate

400ug/ day TO decrease the risk of neural tube defect.

¢  Fetal —  —  — 

if with hypertension. wellbeing tests.

NST beginning at 28 – 30 weeks If non reactive do CST or BPS If clinical conditions deteriorates as in preeclampsia or rapidly developing polyhydramnios – deliver even if with reassuring test.

QUESTIONS: CONTRACEPTION: ¢  Estrogen

– Progesterone –contraindicated if with vascular involvement

¢  Progestin

only & implants

¢  IUD ¢  Barrier

method

¢  Sterilization

 

¢  1.36y/o

G3P2 PU 33 weeks has PPROM for 8 hours.She delivered after 24 hours of labor.On the third postpartum day she developed vaginal bleeding,fever and hypogastric pain.Cervix tender on wriggling,uterus enlarged to 5 months size and tender.What is the diagnosis? ¢  a.cystitis ¢  b.endometritis ¢  c.pyelonephritis ¢  d.thrombophlebitis

¢  2.What

is the most important factor for the development of genital tract infection during puerperium ? ¢  a.number of cervical examination ¢  b.route of delivery ¢  c.length of labor ¢  d.anemia

¢  3.34y/o

G3P3 post CS for 1 week due to prolonged labor complained of vaginal bleeding,abdominal pain and foul smelling discharge.What is the BEST antibiotic management ? ¢  a.ampicillin and gentamycin ¢  b.broad spectrum cephalosporin ¢  c.clindamycin and gentamycin ¢  d.meropenem

¢  4.What

is the microorganism implicated in Toxic Shock syndrome? ¢  a.staphylococcus aureus ¢  b.streptococcus pyogenes ¢  c.Escherichia Coli ¢  d.Pseudomonas

¢  5.25y/oG1P0

¢  6.What

is the best mode of Delivery for a 21y/o G1P0 with RHD mitral stenosis? ¢  a.NSD under sedation ¢  b.assisted vaginal under pudendal ¢  c.forceps extraction under epidural ¢  d.cesarean section

¢  7.A

G3P3 asthmatic patient delivered to a live baby .Which of the following should NOT be given postpartum? ¢  a.antibiotics ¢  b.hydrocortisone ¢  c.terbutaline ¢  d.ergonovine

¢  8.Which

¢  9.23

¢  10.32y/oG2P1

¢  11.What

y/o G4P1 PU 21 weeks has an asymptomatic UTI.Urinalysis showed plenty of pus cells however Urine culture is negative.What is the microorganism implicated? ¢  a.E. Coli ¢  b.chlamydia ¢  c.pseudomonas ¢  d.bacterial vaginosis

PU 35 weeks has recurrent UTI and complains of fever,upper back pain,nausea and vomiting.What is the cornerstone in the management of this patient ? ¢  a.request for creatinine ¢  b.empiric antibiotics ¢  c.hydration with IVF ¢  d.antipyretic

PU 12 weeks with RHD is comfortable at rest but complains of dyspnea while washing the dishes or even when brushing her teeth.What is the new York classification of this patient? ¢  a.1 ¢  b.II ¢  c.III ¢  d.IV

of the following anti TB medications is contraindicated during pregnancy? ¢  a.streptomycin ¢  b.rifampicin ¢  c.pyrazinamide ¢  d.ethambutol

is/ are the laboratory tests needed to evaluate a patient with thyroid disease? ¢  a.MRI ¢  b.thyroid ultrasound ¢  c.TSH ,FT3FT4 ¢  d.thyroid scan

¢  12.23y/o

G1P0 PU 16 weeks has diffuse thyroid enlargement with exopthalmos.TSH is low while FT4 is elevated.What is the BEST treatment for this patient? ¢  a.propanolol ¢  b.iodine ¢  c.prophylthiuracil ¢  d.thyroxine

¢  13.When

¢  15.Which

¢  16.Which

of the following is NOT recommended in patients with Overt DM? ¢  a.alpha feto protein at 16-20 weeks ¢  b.congenital scan at 18-20 weeks ¢  c.weekly doppler velocimetry ¢  d.regular ultrasound for growth

is the recommended age of gestation to screen for gestational DM based on American College of OB –GYN? ¢  a.first trimester ¢  b.16-20 weeks ¢  c.24-28 weeks ¢  d.30-34 weeks

¢  14.21

y/o G1P0 has a result of 145 gms/dl in the 50 gms OGCT.What is the next management for this patient? ¢  a.start oral hypoglycemics ¢  b.start insulin ¢  c.Do 100 gms OGTT ¢  d.manage as normal pregnancy

of the following vaccines must be given to all pregnant patient? ¢  a.hepatitis A ¢  b.HPV ¢  c.influenza ¢  d.pneumonia

Williams Obstetrics 24th Edition

INFECTIOUS DISEASES

¢  First

VARICELLA ZOSTER VIRUS ¢  HERPES

ZOSTER OR SHINGLES – Reactivation of varicella Infection ¢  Unilateral dermatomal vesicular eruption with severe pain

Half of Pregnancy FETAL AND NEONATAL VARICELLA INFECTION SYNDROME

¢  CONGENITAL

— 

Chorioretinitis, micropthalmia, cerebral cortical atrophy, growth restriction, hydronephrosis, limb hypoplasia, cicatricial skin lesion

VARICELLA ZOSTER VIRUS

¢  Exposure —  —  — 

Risk Based on Age of Gestation

< 13 weeks – 0.4% 13-20 weeks – Highest risk >20 weeks – None

¢  Before

or During Delivery: Disseminated visceral, CNS disease (FATAL) 5 DAYS BEFORE AND 2 DAYS AFTER DELIVERY

¢  Recommendation

for exposure: Give VARICELLA ZOSTER Immunoglobulin

VARICELLA ZOSTER VIRUS

¢ DIAGNOSIS: —  —  —  —  — 

Clinical Manifestation Tzanck Smear Tissue culture Direct Fluorescent Antibody Testing NAAT (Nucleic Acid Amplification Test )of Amniotic Fluid

VARICELLA ZOSTER VIRUS

¢  MANAGEMENT ¢  MATERNAL — 

—  —  — 

virus A and B ¢  Fever, dry cough and systemic infection ¢  No firm evidence of malformation ¢  Rare transplacental passage ¢  Reports of stillbirth, preterm delivery and abortion in severe cases

¢  VACCINATION:

VIRAL EXPOSURE:

VARIZIG ( Varicella Zoster Immunoglobulin) within 96 hours of exposure up to 10 days

¢  MATERNAL

— 

INFLUENZA

VARICELLA ZOSTER VIRUS

— 

INFECTION:

— 

NOT recommended for pregnant OR within a month following each vaccine dose Not secreted in breastmilk

Isolation Supportive Chest x-ray Acyclovir 500 mg /m2 or 10-15 mg/kg every 8 hours

INFLUENZA

INFLUENZA

¢  RNA

¢  MANAGEMENT:

¢  Influenza

1. 

— 

—  —  — 

¢ VACCINATION —  Inactivated — 

vaccine RECOMMENDED FOR pregnant during influenza season

¢  Fever,

Viral antigen rapid detection Assay POLYMERASE CHAIN REACTION( PCR) (MOST SENSITIVE) Rapid Influenza Diagnostic Test Immunofluorescence Viral Culture

Coryza, Conjunctivitis, Cough ¢  Erythematous maculopapular rash (Face, neck, back, trunk, extremities) ¢  Koplik spots (small white lesion with erythema in oral cavity ¢  Dx: Serology,-RT-PCR ¢  Treatment: Supportive ¢  Complications during pregnancy: Abortion, preterm delivery, low birth weight

RUBEOLA (MEASLES)

INFLUENZA

a. 

¢  DIAGNOSIS: — 

CATEGORY C DRUGS Neuraminidase Inhibitors

b.  c. 

Oseltamivir – 75 mg BID x 5 days within 48 hours of symptoms Zenamivir (Relenza) Peramivir

2. Adamantanes a.  b. 

Amantadine Rimantidine

RUBEOLA (MEASLES) ¢  Prevention:

Pregnant —  — 

Exposure on Non Immune

IV Immunoglobulin 400 mg /kg within 6 days VACCINATION CONTRAINDICATED DURING PREGNANCY but can be given during post partum breastfeeding

RUBELLA ( GERMAN MEASLES)

RUBELLA ( GERMAN MEASLES)   Fetal

¢ 

Clinical S/Sx:

mild, febrile illness w/ a generalized maculopapular rash   arthralgias   arthritis   Lymphadenopathy   Suboccipital   Postauricular   Cervical ◦  Conjunctivitis  

RUBELLA ( GERMAN MEASLES)

effects:

  Abortion

and severe congenital malformations      

1st 12 wks: 80% 13-14 wks: 54% >14 wks: 25%

RUBELLA ( GERMAN MEASLES)

CONGENITAL RUBELLA SYNDROME  Cataracts

and congenital glaucoma ductus arteriosus and peripheral pulmonary artery stenosis  Sensorineural deafness—the most common single defect  Central nervous system defects (Microcephaly, mental retardation)  Pigmentary retinopathy  Purpura  Hepatosplenomegaly /Jaundice  radioluscent bone densities   Patent

GROUP B STREPTOCOCCUS

  Management: ¢ Diagnosis: — 

ELISA IgM – acute/recent infection ¢  4-5 days after onset of rash up to 6 weeks ¢  IgG – post infection (cant differentiate past immunity and recent disease) ¢  1-2 weeks after onset of rash ¢  High Avidity IgG antibodies – indicate an infection at least 2 months in the past ¢ 

 Symptomatic  Droplet

precautions

  Prevention:  MMR

vaccine (Not given during pregnancy) is avoided 1 month after vaccination

 Pregnancy

     

Streptococcus agalactiae Colonize the vagina and rectum Clinical Characteristics:   Asymptomatic to sepsis   Adverse pregnancy outcome: 1.  2.  3.  4.  5.  6.  7. 

preterm labor prematurely ruptured membranes clinical and subclinical chorioamnionitis bacteriuria Pyelonephritis postpartum endometritis Postpartum maternal osteomyelitis and mastitis

GROUP B STREPTOCOCCUS GROUP B STREPTOCOCCUS

ANTIBIOTIC REGIMEN

PATHOGENESIS & TRANSMISSION ¢  CAUSATIVE

CLINICAL MANIFESTATION

AGENT: T. PALLIDUM

¢  Primary — 

¢  ABRASION

SEXUALLY TRANSMITTED DISEASES Willams 24th Edition

on the vagina (Portal of Entry) & disseminate through lymphatic channels ¢  Incubation period 3-90 days (ave: 3 weeks) ¢  Fetal transmission: Transplacental, contact through lesions of delivery

— 

Elizabeth Ahyong-Reyes, FPOGS

CLINICAL MANIFESTATION

CLINICAL MANIFESTATION

¢  Secondary —  — 

— 

Syphilis

Develops 4-10 weeks after chancre Diffuse macular rash, palmar target like lesions, patchy alopecia, mucous patches Condyloma lata (perineum)- flesh colored papules & nodules.

CLINICAL MANIFESTATION ¢  Tertiary

— 

or Late Syphilis

Slowly progressive disease affecting any organ but rarely seen in reproductive age woman

¢  Secondary

Syphilis

Fever, malaise, anorexia, headache, myalgias, arthralgias —  Rarely: aseptic meningitis, hepatitis, nephropathy, ocular changes, anterior uveitis, periostitis

Syphilis

Painless chancre with raised red firm border and smooth base, non suppurative lymphodenopathy Resolves in 2-8weeks even if untreated

CLINICAL MANIFESTATION ¢  Latent

Syphilis

— 

CONGENITAL INFECTION Hepatic Abnormalities Anemia Thrombocytopenia Ascites & Hydrops STILLBIRTH

(+) serological testing with no clinical manifestation —  Early latent – within 12 weeks —  Late latent - >12 months — 

NEWBORN ¢  Jaundice,

purpuric skin lesion ¢  Lymphadenopathy ¢  Rhinitis ¢  Pneumonia ¢  Myocarditis ¢  Nephropathy ¢  Long bone involvement

PLACENTA ¢  Large

and pale ¢  Chorionic villi: lose arborization, thicker and clubbed ¢  Diminish blood vessels

DIAGNOSIS

PERINATAL DIAGNOSIS

¢  Definitive

1. 

Diagnostic: Dark field Examination & Direct Immunoflurescent Antibody ¢  Screening Test: 1. Non Treponemal VDRL, RPR 2. Treponemal - Specific FTA – ABS, MHA – TP, TP – PA (Trep. Pallidum – Particle Agglutination)

Ultrasound: hydrops, ascites, hepatomegaly and placental thickening, hydramnios

2. Doppler Velocimetry – elevated in middle cerebral artery 3. PCR of AF / Darkfield Illumination of AF

TREATMENT

PENICILLIN ALLERGY

GONORRHEA

¢  History

of Allergy è oral stepwise penicillin dose è challenge or skin testing ¢ 

Penicillin Desensitization

¢  JARISCH

Herxheimer Reaction – uterine contraction, FHR deceleration

GONORRHEA

TREATMENT

¢  Marker

of concomitant chlamydial infection ¢  Limited to lower genital tract ¢  COMPLICATIONS: septic abortion, preterm delivery, PPROM, chorioamnionitis, Postpartum infection

GONORRHEA

1. SCREENING Culture NAATC (Nucleic acid Amplification Test)

NEWBORN: Erythromycin, Tetracycline, Silver Nitrate

CHLAMYDIA ¢  Most

common infectious disease asymptomatic or urethral syndrome, urethritis or Bartholins gland infection, Mucopurulent cervicitis ¢  Perinatal Transmission – Conjunctivitis/ Pneumonia ¢  Mostly

SCREENING & TREATMENT

HERPES SIMPLEX VIRUS

¢  Screening — 

Culture (NAAT)

¢  Treatment

HSV 2

HERPES SIMPLEX VIRUS AGENT: ¢  HSV 1 non genital ¢  HSV 2 Genital

CLINICAL MANIFESTATION

CLINICAL MANIFESTATION

1. First Episode Primary Infection

2. First Episode Non-primary Infection ¢  HSV is isolated with other HSV type antibody ¢  Milder signs and symptoms

¢  CAUSATIVE

¢  NEONATAL

TRANSMISSION 1. Intrauterine (5%) 2. Peripartum (85%) 3. Postnatal (10%)

¢  Papule,

itching and tingling è vesicular and painful ¢  Lymphadenopathy ¢  Transient Influenza like symptoms

3. Reactivation – ¢  Viral shedding of the HSV residing in the nerve ganglia

NEWBORN INFECTION

DIAGNOSIS

MANAGEMENT

¢  Skin,

1. Virological Test ¢  Cell culture & PCR (Preferred Test) ¢  * negative test does not exclude Infection

¢  Acyclovir,

eye, mouth lesions (40%) disease with encephalitis (30%) ¢  Disseminated disease of multiple organ involvement (32%) ¢  Not associated with abortion and stillbirth ¢  CNS

2. Type Specific Serological Test ¢  ELISA or Blot style test for HSV glycoproteins

Famcyclovir, Valacyclovir (decrease duration of symptoms and viral shedding) ¢  Oral Analgesic ¢  Topical Anesthetic ¢  Indwelling catheter for urinary retention

MANAGEMENT

PERIPARTUM SHEDDING PROPHYLAXIS

HUMAN PAPILLOMA VIRUS INFECTION ¢  40

¢  CS

is recommended for women with active genital lesions or prodromal symptoms ¢  CS is not recommended for active lesions in non genital area ¢  No evidence that external lesions causes ascending infection in PPROM ¢  Breastfeeding

is allowed as long as there no

—  — 

types infect genital tract

Type 16 & 18 oncogenic and high risk to dysplasia Type 6 & 11 mucocutaneous external genital wart

¢  Increase

in number to size during pregnancy

breast lesions

TREATMENT ¢  80-90%trichloracetic

acid – topically once a week ¢  Cryotherapy, laser or surgical excision ¢  Contraindicated: podophyllin, podofilox, imiquimod, interferon, sinecathechines ¢  Laryngeal papillomatosis is rare. CS is no longer indicated to decrease transmission risk

TREATMENT ¢ VACCINATION —  Quadrivalent-Gardasil —  Bivalent-Cervarix

NOT given during pregnancy but can be given to breastfeeding women.

WHAT AFFECT RAPIDITY OF PROGRESSION:

¢  Causative

1. 

—  —  — 

Blood or blood contaminated products Perinatal breastfeeding

¢  Primary

method of Dx: HIV I viral load

¢  BACTERIAL —  — 

VAGINOSIS

Not an infection; fishy smelling discharge Decrease lactobacilli and increase anaerobes (Gardnerella, Mobilincus & Bacteroides)

¢  Risk

HUMAN IMMUNODEFICIENCY VIRUS agents: RNA retrovirus HIV 1 (most common) & HIV 2 ¢  Method of transmission

VAGINITIS

2.  3.  4. 

Route of infection Pathogenicity of infecting viral strain Initial viral inoculum Immunological status of the host

factors: drinking, multiple partners, young age, smoking, black race, and vitamin D deficiency ¢  Complications: preterm birth, miscarriage low birth weight, neonatal morbidity ¢  Management: Metronidazole 500mg BID x 7days or 250mg q8hrs x 7 days; Clindamycin 300mg BID x 7days.

CLINICAL SIGNS OF PROGRESSION ¢ Generalized

lymphadenopathy hairy leuplakia ¢ Apthous ulcers ¢ Thrombocytopenia ¢ Oral

DIAGNOSTICS

MATERNAL & PERINATAL TRANSMISSION

¢  SCREENING:

¢  Vertical

immunoassay

enzyme linked

—  — 

¢  CONFIRMATORY:

westernblot or immunoflourescence asssay

— 

MANAGEMENT DURING PREGNANCY

transmission

— 

More common in prolonged membrane rupture Delivery Breastfeeding

—  —  —  —  —  —  — 

PREFERRED CLASSES OF ANTIRETROVIRAL DRUGS DURING PREGNANCY 1.  2.  3.  4.  5.  6. 

Lamivudine Zidovudine Ritonavir Atazanavir Lopinavir/ ritonavir Nevirapine

LABORATORY ASSESSMENT

COMPLICATIONS OF HIV INFECTION

¢  CD4

1. 

¢  HIV

T-lymphocyte count RNA viral load

Pneumocystic jiroveci pneumonia — 

sulfamethoxazole trimetoprim or Dapsone

¢  CBC ¢  Liver

function test testing at 24-48 weeks

¢  Glucose

2. 

PRENATAL HIV TRANSMISSION

PRENATAL HIV TRANSMISSION

¢  Highly

¢  CS

has been recommended to decrease HIV prenatal transmission for patients with 1000 copies/ml of HIV RNA load at 38 weeks ¢  IV Zidovudine as loading dose prior to CS followed by 2 more hours of continuous maintainance therapy

Opportunistic infections — 

3. 

active aniretroviral therapy or HAART –reduced perinatal HIV transmission ¢  Avoid artificial rupture of membranes, invasive fetal monitoring ¢  Avoid forceps or vacuum extraction ¢  Methergine and other ergot adversely interact with antiretroviral drugs causing vasoconstriction

serum creatinine, hemogram, and bacteriuria screening Plasma HIV RNA quantification—“viral load,” CD4+ Tlymphocyte count, and antiretroviral resistance testing Serum hepatic aminotransferase levels HSV-1 and -2, cytomegalovirus, toxoplasmosis, and hepatitis C serology screening chest radiograph Tuberculosis skin testing—purified protein derivative (PPD) Pneumococcal, hepatitis B, hepatitis A, Tdap, and influenza vaccines Sonographic evaluation

Toxoplasmosis, herpes, mycobacteria, candida

IUGR, preeclampsia, PPROM

HEMATOLOGICAL DISORDERS OF PREGNANCY

ANEMIA ¢  hemoglobin

concentration less than 12 g/dL in nonpregnant women and less than 10 g/dL during pregnancy or the puerperium

¢  The

Centers for Disease Control and Prevention (1990) defined anemia as less than 11 g/dL in the first and third trimesters and less than 10.5 g/dL in the second trimester

IRON-DEFICIENCY ANEMIA • 

–  –  –  • 

ETIOLOGIES: —  —  —  — 

— 

Abortion ectopic pregnancy hydatidiform mole placenta previa

–  – 

—  — 

Whole blood transfusion to maintain perfusion to vital organs hemostasis iron therapy for at least 3 mos

erythrocyte hypochromia and microcytosis low serum ferritin levels non stainable bone marrow iron

— 

EFFECTS ON THE MOTHER: —  — 

— 

— 

• 

ETIOLOGY: – 

• 

consequence of antibodies directed against platelets

DIAGNOSIS: – 

– 

• 

EFFECTS ON THE FETUS: – 

– 

MANNER OF DELIVERY:

• 

MANAGEMENT:

– 

–  –  – 

benign condition hemorrhagic complication

most common malignancy of women of all age groups

thrombocytopenia inc risk of intracranial hemorrhage

• 

EFFECTS ON THE MOTHER: – 



intrapartum fetal scalp platelet determinations once the cervix was 2 cm dilated and the membranes ruptured percutaneous umbilical cord blood sampling

ferrous sulfate, fumarate, or gluconate 200 mg daily of elemental iron transfusion of pRBC if hgb
View more...

Comments

Copyright ©2017 KUPDF Inc.
SUPPORT KUPDF